Está en la página 1de 83

Breve Introducción

a la Resolución de Problemas

Francisco Javier Garcı́a Capitán

Material impartido en los cursos


Thales-CICA 2007-08 y 2008-09, a cargo de
Ricardo Barroso Campos (Universidad de Sevilla) y
Francisco Javier Garcı́a Capitán (I.E.S. Álvarez Cubero).
2
Contenido

1. Introducción 7
1.1. Fases en las resolución de un problema . . . . . . . . . . . . . 7
1.2. ¿Cómo empezar? . . . . . . . . . . . . . . . . . . . . . . . . . 7
1.2.1. Busca un patrón . . . . . . . . . . . . . . . . . . . . . 8
1.2.2. Haz una figura . . . . . . . . . . . . . . . . . . . . . . 8
1.2.3. Busca simetrı́as . . . . . . . . . . . . . . . . . . . . . . 9
1.2.4. Ve hacia atrás . . . . . . . . . . . . . . . . . . . . . . . 9
1.2.5. Considera la paridad . . . . . . . . . . . . . . . . . . . 10
1.3. Métodos de demostración . . . . . . . . . . . . . . . . . . . . . 10
1.3.1. Demostración directa . . . . . . . . . . . . . . . . . . . 11
1.3.2. Considerar diferentes casos . . . . . . . . . . . . . . . . 11
1.3.3. Demostración mediante un contraejemplo . . . . . . . . 12
1.3.4. Demostración por reducción al absurdo . . . . . . . . . 12
1.3.5. El método de inducción . . . . . . . . . . . . . . . . . 12
1.3.6. El principio del palomar . . . . . . . . . . . . . . . . . 14
1.4. Problemas propuestos en la edición anterior . . . . . . . . . . 15
1.5. Problemas propuestos . . . . . . . . . . . . . . . . . . . . . . . 17

2. Simetrı́a 19
2.1. Introducción . . . . . . . . . . . . . . . . . . . . . . . . . . . . 19
2.2. Problemas geométricos . . . . . . . . . . . . . . . . . . . . . . 20
2.3. Problemas no geométricos . . . . . . . . . . . . . . . . . . . . 21
2.4. Problemas de la edición anterior . . . . . . . . . . . . . . . . . 24
2.5. Problemas propuestos . . . . . . . . . . . . . . . . . . . . . . . 26

3. Desigualdades 27
3.1. Medias . . . . . . . . . . . . . . . . . . . . . . . . . . . . . . . 27
3.2. La desigualdad A-G . . . . . . . . . . . . . . . . . . . . . . . . 27
3.2.1. Demostración por inducción . . . . . . . . . . . . . . . 27
3.2.2. Demostración por sustituciones . . . . . . . . . . . . . 29

3
4 CONTENIDO

3.2.3. Problemas resueltos . . . . . . . . . . . . . . . . . . . . 29


3.3. Desigualdad entre las medias geométrica y armónica . . . . . . 30
3.4. La desigualdad de Cauchy-Schwarz . . . . . . . . . . . . . . . 31
3.4.1. La desigualdad de Cauchy-Schwarz . . . . . . . . . . . 31
3.4.2. Problemas resueltos . . . . . . . . . . . . . . . . . . . . 31
3.5. La desigualdad de Chebyshev . . . . . . . . . . . . . . . . . . 32
3.6. Problemas de la edición anterior . . . . . . . . . . . . . . . . . 34
3.7. Problemas propuestos . . . . . . . . . . . . . . . . . . . . . . . 36

4. Geometrı́a del triángulo 37


4.1. Notación en el triángulo . . . . . . . . . . . . . . . . . . . . . 37
4.2. Teoremas de los senos y del coseno . . . . . . . . . . . . . . . 37
4.3. Área de un triángulo . . . . . . . . . . . . . . . . . . . . . . . 38
4.4. El teorema de la bisectriz . . . . . . . . . . . . . . . . . . . . 39
4.5. El teorema de Stewart . . . . . . . . . . . . . . . . . . . . . . 40
4.6. El teorema de Ceva . . . . . . . . . . . . . . . . . . . . . . . . 42
4.7. El teorema de Menelao . . . . . . . . . . . . . . . . . . . . . . 44
4.8. Problemas de la edición anterior . . . . . . . . . . . . . . . . . 45
4.9. Problemas propuestos . . . . . . . . . . . . . . . . . . . . . . . 49

5. Coordenadas baricéntricas 51
5.1. Geometrı́a de masas . . . . . . . . . . . . . . . . . . . . . . . 51
5.1.1. Introducción . . . . . . . . . . . . . . . . . . . . . . . . 51
5.1.2. Propiedades . . . . . . . . . . . . . . . . . . . . . . . . 51
5.1.3. Problemas resueltos . . . . . . . . . . . . . . . . . . . . 52
5.2. Coordenadas baricéntricas . . . . . . . . . . . . . . . . . . . . 53
5.2.1. Coordenadas baricéntricas homogéneas . . . . . . . . . 53
5.2.2. Problemas resueltos . . . . . . . . . . . . . . . . . . . . 54
5.3. Area de un triángulo . . . . . . . . . . . . . . . . . . . . . . . 55
5.4. Rectas . . . . . . . . . . . . . . . . . . . . . . . . . . . . . . . 56
5.4.1. Ecuación de la recta que pasa por dos puntos . . . . . 56
5.4.2. Puntos del infinito . . . . . . . . . . . . . . . . . . . . 57
5.4.3. Rectas paralelas . . . . . . . . . . . . . . . . . . . . . . 58
5.4.4. Intersección de dos rectas . . . . . . . . . . . . . . . . 58
5.5. Problemas de la edición anterior . . . . . . . . . . . . . . . . . 59
5.6. Problemas propuestos . . . . . . . . . . . . . . . . . . . . . . . 62

6. Ecuaciones funcionales 63
6.1. Ecuaciones funcionales . . . . . . . . . . . . . . . . . . . . . . 63
6.2. La ecuación f (x + y) = f (x) + f (y) . . . . . . . . . . . . . . . 63
6.3. Técnicas de resolución . . . . . . . . . . . . . . . . . . . . . . 64
CONTENIDO 5

6.4. Problemas resueltos . . . . . . . . . . . . . . . . . . . . . . . . 65


6.5. Problemas de la edición anterior . . . . . . . . . . . . . . . . . 68
6.6. Problemas propuestos . . . . . . . . . . . . . . . . . . . . . . . 70

A. Soluciones 71
6 CONTENIDO
Tema 1
Introducción

1.1. Fases en las resolución de un problema


George Polya (1887-1985) en su libro How solve it señaló las siguientes
fases a la hora de resolver un problema:

Comprender bien el enunciado. Debemos entender perfectamen-


te el problema planteado, distinguiendo la incógnita y los datos del
problema.

Diseñar un plan. Debemos encontrar un camino desde los datos hasta


la incógnita, basándonos en los conocimientos adquiridos de la teorı́a, de
la resolución de otros problemas. Nos podemos ayudar de un esquema
o dibujo, de si la situación presenta alguna simetrı́a. También podemos
cambiar un poco el problema para intentar resolver antes una versión
simplificada, etc.

Llevar a cabo el plan. Ahora habrı́a que recorrer el camino en-


contrado en el apartado anterior. Si nos encontramos con dificultades
habrá que replantearse las conexiones entre los datos y la incógnita, o
buscar otros nuevos.

Reflexionar sobre la solución obtenida. Debemos analizar si la so-


lución obtenida se corresponde exactamente con lo que pedı́a el proble-
ma. También podemos aplicar esa solución a resolver otros problemas
parecidos, o inventar nuevos problemas a partir del resuelto.

1.2. ¿Cómo empezar?


Muchas veces, al principio sobre todo, la primera pregunta que nos hare-
mos cuando leamos el enunciado de un problema será: ¿por donde empiezo?

7
8 TEMA 1. INTRODUCCIÓN

Esta sección intenta contestar a esta pregunta mediante algunas estrategias


que podemos usar para comenzar a atacar un problema.

1.2.1. Busca un patrón


Una buena forma de convencerse de que la cuestión planteada por un
problema puede resolverse es comprobar que se cumple en algunos casos
especiales. Por ejemplo, consideremos el siguiente problema:

Problema 1. Calcular el cuadrado de 333.333.333.334.


Este número está formado por varios treses y un cuatro al final, ası́ que
podemos ver lo que ocurre con los números 34, 334, 3334, etc:

34 · 34 =1156,
334 · 334 =111556,
3334 · 3334 =11115556,
···

Parece que el resultado siempre está formado por unos (uno más que
treses), cincos (igual número que treses) y un 6 al final. Ya hemos encontrado
un patrón. Ahora habrá que demostrar que esto se cumple en general, pero
ya sabemos lo que hay que demostrar.

1.2.2. Haz una figura


Es habitual que hagamos una figura para comprender mejor el enunciado
y para comenzar a resolver un problema geométrico.
El uso de figuras también es posible y útil en problemas que no son
geométricos. Como ejemplo, consideremos el siguiente problema.

Problema 2. Tres parejas se encuentran para salir. Algunos se saludan con


un beso. Uno de ellos, Juan, pregunta a cada uno de los otros con cuántos
se besó, y todos le responden números distintos. Si se sabe que ninguno se
besó con su propia pareja, ¿con cuántas personas se besó Juan?
Solución. Como ninguno besó a los otros cinco, las respuestas distintas
son 0, 1, 2, 3, 4. Representamos por J a Juan, y por los números 0, 1, 2, 3,
4 a las personas, según el número de besos que dio cada una. Entonces, 4
tuvo que besar a Juan, a 1, a 2 y a 3. 1 sólo se besó con 4. 3 tuvo que besar
a 2, 4 y Juan. 2 y Juan ya no besaron a nadie más. Todo esto lo podemos
dibujar con el siguiente gráfico:
1.2. ¿CÓMO EMPEZAR? 9

2
0 1 4 3
J
Figura 1
Teniendo en cuenta que nadie besa a su pareja, son parejas 0 y 4, 1 y 3,
y 2 y Juan. Juan se besó con 2 personas: 3 y 4.

1.2.3. Busca simetrı́as


Es habitual encontrarse simetrı́as en problemas geométricos. Por ejemplo,
consideremos éste:
Problema 3. En el triángulo rectángulo ABC dibujamos un semicı́rculo
con diámetro BE sobre el lado AB y tangente a la hipotenusa AC en D. Si
AE = 3 y BC = 9, ¿cuál es el diámetro del semicı́rculo?
A

3
E
D

9
B C
Figura 2
Si completamos el triángulo ABC con su simétrico respecto de la rec-
ta AB, la semicircunferencia se transformará en la circunferencia inscrita a
un triángulo y podremos usar conceptos que sepamos de la circunferencia
inscrita.

1.2.4. Ve hacia atrás


Hay veces en las que, habiendo designando una incógnita, si vamos plan-
teando el problema con el enunciado desde el principio al final, llegamos a una
expresión demasiado complicada. Sin embargo, si empezamos por el final, el
resultado puede ser más sencillo. Como muestra, consideremos el siguiente
ejemplo:
10 TEMA 1. INTRODUCCIÓN

Problema 4. Una campesina llevó a la ciudad una cesta de huevos. Al pri-


mer cliente le vendió la mitad de sus huevos más medio huevo. Al segundo
cliente le vendió la mitad de los huevos que le quedaban mas medio huevo.
Al tercer cliente le vendió la mitad de los huevos que le quedaban más medio
huevo. Si le quedaban tres huevos, en la cesta, ¿cuántos llevaba al principio?

Solución. Sean z los huevos que le quedaban


( ) a la campesina antes de
vender huevos al tercer cliente. De z − z2 + 12 = 3 deducimos que z = 7
huevos.

Ahora, sean y los huevos que le( quedaban


) a la campesina antes de vender
huevos al segundo cliente. De y − 2 + 2 = 7 deducimos que y = 15 huevos.
y 1

Por último, sean x los huevos que le( quedaban


) a la campesina antes de
vender huevos al primer cliente. De x − x2 + 12 = 15 deducimos que x = 31
huevos.

1.2.5. Considera la paridad


Algunos problemas pueden resolverse considerando las propiedades par e
impar de cierta cantidad. Por ejemplo,

Problema 5. Los números del 1 al 10 se escriben en una lı́nea. ¿Podemos


intercalar los signos + y − entre ellos de manera que la operación resultante
tenga valor cero?

Solución. Imaginemos que hemos puesto adecuadamente los signos + y


− y que el resultado es 0. Entonces habremos descompuesto los números del
1 al 10 en dos grupos A y B de igual suma. Pero los números del 1 al 10
suman 55, ası́ que los números de A y los números de B tendrı́an que sumar
27.5 cada uno, lo cual es imposible. La idea clave ha sido que los números
del 1 al 10, que suman un número impar, no pueden descomponerse en dos
grupos de igual suma.

1.3. Métodos de demostración


Algunas veces la resolución de un problema consiste en demostrar la ve-
racidad o falsedad de un enunciado. La demostración debe ser lo suficiente
explı́cita para convencer a la persona a la que va dirigida. A continuación
exponemos algunos métodos de demostración.
1.3. MÉTODOS DE DEMOSTRACIÓN 11

1.3.1. Demostración directa


Consideremos el siguiente problema:

Problema 6. Demostrar que si n es un número par, entonces n2 es múltiplo


de 4.
Solución. Efectivamente, si n es par, entonces n = 2k para algún entero
k. En consecuencia, elevando al cuadrado, será n2 = 4k 2 , es decir n2 es un
múltiplo de 4.
Hemos hecho un razonamiento que comienza con la afirmación de que n
es un número par y acaba con la afirmación de que n2 es múltiplo de 4.
Las demostraciones, como ésta, en las que partimos de un enunciado (la
hipótesis, los datos del problema) y llegamos a otro enunciado (la tesis, la
incógnita del problema) se llaman demostraciones directas.
Veamos otro ejemplo:

Problema 7. El área de un triángulo rectángulo ABC de hipotenusa c tiene


2
área c4 . Demostrar que el triángulo es isósceles.
Solución. Llamamos x e y a los catetos del triángulo. Queremos demostrar
2
que x = y suponiendo que xy2
= c4 . Entonces,

xy c2 xy x2 + y 2
= ⇒ = ⇒ x2 + y 2 = 2xy ⇒
2 4 2 4
⇒x2 + y 2 − 2xy = 0 ⇒ (x − y)2 = 0 ⇒ x = y.

El camino desde los datos hasta la incógnita se recorre haciendo uso de


conocimientos previos, como la fórmula del área de un rectángulo, el teorema
de Pitágoras y la factorización de una expresión.

1.3.2. Considerar diferentes casos


Problema 8. Demostrar que n(n + 1)(2n + 1) es divisible por 6.
Solución. Por un lado n(n + 1) siempre es par, ası́ que también lo es
n(n + 1)(2n + 1). Ahora, si alguno de los números n o n + 1 fuera divisible
por 3, también lo serı́a n(n + 1) y por supuesto n(n + 1)(2n + 1). Y en caso
contrario, si ninguno de los números consecutivos n y n + 1 es divisible por 3,
es seguro que sı́ lo serán tanto n − 1 como n + 2, y por tanto también su suma
(n − 1) + (n + 2) = 2n + 1, resultando que n(n + 1)(2n + 1) es igualmente
un múltiplo de 3. Finalmente, como n(n + 1)(2n + 1) es múltiplo de 2 y de
3, también es múltiplo de 6.
12 TEMA 1. INTRODUCCIÓN

1.3.3. Demostración mediante un contraejemplo

Para demostrar que un enunciado es falso es suficiente comprobar que


no es cierto para un caso particular. Por ejemplo, no cuesta encontrar unos
valores x e y tales que (x + y)2 ̸= x2 + y 2 , lo que nos dice que la igualdad
(x + y)2 = x2 + y 2 no es una identidad.
Otro ejemplo es el de la ecuación a4 +b4 +c4 = d4 . Euler (Leonhard Euler,
1707-1783) conjeturó que no tendrı́a ninguna solución con valores enteros. Sin
embargo, mediante el empleo de ordenadores se ha encontrado recientemente
que 958004 + 2175194 + 4145604 = 4224814 , por lo que podemos decir que el
enunciado asegurado por Euler era falso.

1.3.4. Demostración por reducción al absurdo


La demostración por reducción al absurdo de que un enunciado A implica
un enunciado B se hace suponiendo que la conclusión B es falsa y llegando
a una contradicción con lo afirmado por A.
Por ejemplo consideremos el siguiente problema:

Problema 9. Demostrar que 2 es irracional.
√ √
Solución. Supongamos que 2 es racional, y que 2 = m n
(fracción irredu-
2 2
cible). Entonces m = 2n y el número m debe ser par, es decir, será m = 2p
para algún entero p. Pero entonces, 4p2 = m2 = 2n2 ⇒ n2 = 2p2 , por lo que
n también debe ser par, lo cual es imposible, ya que habı́amos supuesto que
la fracción m
n
era irreducible.

1.3.5. El método de inducción


Imaginemos infinitas fichas de dominó (Figura 3) puestas de pie sobre
sobre una mesa infinitamente larga.
Supongamos que ocurren estas circunstancias:

1. Un dedo inquieto se acerca a la primera ficha y la derriba.

2. Si cualquiera de las fichas cae, la siguiente también cae.

¿Qué ocurrirá con las infinitas fichas? No hay duda. Caerán todas. Este
es el principio en el que se basa el método de inducción.
1.3. MÉTODOS DE DEMOSTRACIÓN 13

Figura 3

Consideremos un enunciado que depende de un número natural n cual-


quiera. Por ejemplo, la validez de la fórmula

n(n + 1)
1 + 2 + ··· + n = .
2
Podemos imaginar que cuando demostramos que la fórmula es cierta para
un cierto valor de n es una ficha que cae. Todo lo que tenemos que hacer es
demostrar que la fórmula es cierta para n = 1 (equivalente a hacer caer la
primera ficha) y, a continuación, demostrar que si la fórmula es cierta para
un n también lo es para el siguiente. Con ello, todas las fichas caerán, es
decir la fórmula será cierta para todos los n. Veamos como se hace esto en
este ejemplo:

1. Comprobamos que la fórmula es cierta para n = 1.

1 · (1 + 1)
1 = S1 = .
2

2. Demostramos que si la fórmula es cierta para n, también lo es para


n + 1. Es decir, supondremos que

n(n + 1)
1 + 2 + ··· + n =
2
y demostraremos que

(n + 1)(n + 2)
1 + 2 + · · · + (n + 1) = .
2
14 TEMA 1. INTRODUCCIÓN

En efecto,
1 + 2 + · · · + n + (n + 1) =
= [1 + 2 + · · · + n] + (n + 1) =
n(n + 1)
= + (n + 1) =
2 ( )
n
=(n + 1) +1 =
2
(n + 1)(n + 2)
= .
2

1.3.6. El principio del palomar


El principio del palomar o principio de Dirichlet (Johann Peter Gustav
Lejeune Dirichlet, 1805-1859) afirma

Si n + 1 palomas se introducen en un palomar con n nidos, tiene


que haber al menos un nido con más de una paloma.

Podemos resolver muchos problemas si identificamos correctamente en el


enunciado cuáles son las palomas y cuales son los nidos. Comencemos con el
siguiente problema:

Problema 10. Dentro de un grupo de 13 amigos, seguro que hay dos que
nacieron el mismo mes del año.
En este caso, el planteamiento es sencillo, pues tenemos 13 palomas (los
amigos) y 12 nidos (los meses de año).
Otras veces, determinar cuáles son las palomas y cuáles los nidos no es
tan sencillo:

Problema 11. Un equipo de baloncesto juega durante el mes de Junio, al


menos un partido diario y no más de 45 partidos en total. Demostrar que en
un determinado perı́odo de dı́as consecutivos el equipo jugó exactamente 14
partidos.
Solución. Llamemos aj al número de partidos que jugó el equipo hasta
el dı́a j de Junio incluido. Los números a1 , a2 , . . . , a30 forman una sucesión
estrictamente creciente de enteros entre 1 y 45. Además, los números a1 +
14, a2 + 14, . . . , a30 + 14 es una sucesión estrictamente creciente de números
enteros entre 15 y 59. En total, son 60 números entre 1 y 59. Los números del
1 al 59 son los nidos y los 60 números de las dos sucesiones son las palomas.
Por el principio del palomar, entre los sesenta números debe haber al
menos dos iguales. Uno de los primeros debe ser igual a uno de los segundos,
1.4. PROBLEMAS PROPUESTOS EN LA EDICIÓN ANTERIOR 15

es decir aj = ak + 14 para algunos dı́as j y k. Como conclusión, entre los dı́as


j y k, el equipo jugó 14 partidos.
Problema 12. Dados 27 números impares menores que 100, demostrar que
hay al menos un par de ellos cuya suma es 102.
Solución. Hay 50 números impares menores que 100, y 48 de ellos for-
man parejas cuya suma es 102: (3, 99), (5, 87), . . . , (49, 53). Cada una de estas
parejas será uno de nuestros nidos. Además consideramos otros dos nidos,
formado cada uno de ellos por el los números que no tienen pareja: 1 y 51.
En total hay 26 nidos. Como tenemos 27 números (las palomas), dos de ellos
serán del mismo nido, y tiene que ser de uno de los 24 primeros, ası́ que esos
dos números sumarán 102.

1.4. Problemas propuestos en la edición an-


terior
Problema 13. Demostrar que si el número a es racional no nulo y el número
b es irracional, entonces el número ab es irracional.
Solución. Razonamos por reducción al absurdo. Supongamos que a es
racional y b es irracional, y supongamos que el número c = ab es racional.
Entonces, despejando, b = ac también serı́a racional, lo cual es contradictorio
con lo supuesto.
Problema 14. Demostrar la validez de la fórmula
n3 n2 n
12 + 22 + · · · + n2 = + + .
3 2 6

Solución. Usamos el método de inducción. Llamamos Sn a la suma 12 +


22 + · · · + n2 . Para n = 1, se cumple que S1 = 1 = 21 + 13 + 16 , por lo que la
fórmula es cierta para n = 1. Ahora suponemos que la fórmula es cierta para
n, es decir, suponemos que se cumple que
n3 n2 n
Sn = + + ,
3 2 6
que demostraremos que la fórmula se cumple para n + 1, es decir,
(n + 1)3 (n + 1)2 n + 1
Sn+1 = + + =
3 2 6
n3 3 13
= + n2 + n + 1.
3 2 6
16 TEMA 1. INTRODUCCIÓN

Comenzamos:
Sn+1 =12 + 22 + · · · + n2 + (n + 1)2 =
( )
= 12 + 22 + · · · + n2 + (n + 1)2 =
n3 n2 n
= + + + n2 + 2n + 1 =
3 2 6
n3 3 2 13
= + n + n + 1.
3 2 6
Problema 15. Demostrar que, para cualquier n, 7n − 1 es divisible por 6.
Solución. Razonamos por inducción. Para n = 1, 7n − 1 = 71 − 1 = 6,
que es divisible por 6. Suponiendo que 7n − 1 es divisible por 6, podemos
expresar
7n+1 − 1 = 7 · 7n − 1 = 6 · 7n + (7n − 1),
que es la suma de dos números divisibles por 6, también divisible por 6.
Problema 16. Probar que en cualquier conjunto de seis personas forzosa-
mente hay tres que se conocen todas entre sı́ o tres tales que ninguna conoce
a ninguna de las otras dos.
Solución. Sean A, B, C, D, E, F las seis personas.
Supongamos que A conoce a tres (o más) de los otros, por ejemplo B, C
y D. Si B y C, o B y D, o C y D se conocen, A y esa pareja formarı́an tres
personas que se conocen las tres. En caso contrario B, C y D forman un trio
de tres personas que no se conocen.
Supongamos que A conoce como mucho a dos personas, por ejemplo B y
C. Si D y E, o E y F , o D y F no se conocen, A y esa pareja suman tres
personas que no se conocen ninguna a la otra. En caso contrario, D, E y F
son tres personas que se conocen.
Problema 17. Demostrar que en un grupo de 50 personas hay dos personas
que tienen el mismo número de amigos (se supone que la amistad es una
relación mutua).
Solución. La única posibilidad para que cada una de las personas sea
amiga de un número diferente de personas es que, sin pérdida de generalidad,
la primera no sea amiga de nadie, la segunda sea amiga de una persona, la
tercera de dos, y ası́ hasta la última, que debe ser amiga de 49 personas.
Ahora bien, es incompatible que haya una persona que es amiga de todas las
restantes con que haya una persona que no sea amiga de nadie. Por tanto,
tendrı́amos un caso del principio del palomar con 50 palomas (las personas)
y 49 o menos nidos (los amigos que puede tener una persona en el grupo).
1.5. PROBLEMAS PROPUESTOS 17

1.5. Problemas propuestos

1.1. Encontrar el fallo en el siguiente razonamiento de que todas las personas


tienen el mismo peso:

Sea P (n) la afirmación de que en cualquier grupo de n perso-


nas, todas tienen el mismo peso. Veamos que P (n) es cierta para
todos los n. Evidentemente P (1) es cierta, ya que si solo hay
una persona en el grupo, todas las personas del grupo tendrán
el mismo peso. Suponiendo que P (n) es cierta demostremos que
P (n + 1) también es cierta. Para ello consideramos un grupo de
n + 1 personas p1 , p2 , . . . , pn+1 . Queremos comprobar que todas
ellas tienen el mismo peso. Para ello, consideremos los dos grupos
de n personas resultantes de apartar a la primera persona y a la
última, es decir los grupos formados por las personas p1 , . . . , pn y
p2 , . . . , pn+1 , respectivamente. En cada grupo, por tener n perso-
nas, cada una de ellas tendrá el mismo peso, y como, por ejem-
plo, p2 es común a ambos grupos, todas las personas del grupo
p1 , p2 , . . . , pn+1 tienen el mismo peso.

1.2. Demostrar que si a, b son enteros tales que 7 divide a a + b y a2 + b2 ,


entonces 7 divide a a y a b.
1.3. Demostrar la regla de divisibilidad por 11.
1.4. Un ordenador ha sido usado durante 99 horas en un periodo de 12
dı́as. Sabiendo que el primer dı́a el ordenador se usó 7 horas, y el último dı́a
6 horas, demostrar que en algún par de dı́as consecutivos el ordenador fue
usado al menos 17 horas.
1.5. Demostrar que si x+ x1 es un entero, también lo es xn + x1n para cualquier
entero n ≥ 1.
18 TEMA 1. INTRODUCCIÓN
Tema 2
Simetrı́a

2.1. Introducción
La simetrı́a es un concepto matemático importante, que juega un papel
fundamental cuando es usado como técnica de resolución de problemas. Sin
embargo, rara vez es usado en la enseñanza secundaria para resolver proble-
mas matemáticos.
En las páginas que siguen veremos cómo aplicar la simetrı́a a problemas
geométricos y a otros que no lo son, y donde el uso de la simetrı́a no es
evidente.
Aunque la idea de simetrı́a es intuitiva, puede ser útil contar con una
definición más formal: podemos decir que simetrı́a es una terna formada por
un objeto S, una propiedad p del objeto y una transformación T cumpliendo
las condiciones siguientes:
1. El objeto pertenece al dominio de la transformación.
2. La aplicación del la transformación al objeto no cambia la propiedad
del objeto.
Como dice Polya, esperamos que la simetrı́a que encontremos en los datos
de un problema se vean reflejados en la solución. Esto es lo que expresa el
principio de razón insuficiente, que puede enunciarse ası́:

Si no hay suficientes razones para hacer distinciones, entonces no


las hacemos.

Por ejemplo, consideremos el siguiente problema:


Problema 18. Hallar el valor máximo de sen A + sen B + sen C siendo A,
B y C los ángulos de un triángulo.
No hay ninguna razón para dar a ninguno de los ángulos mayor o me-
nor importancia que a los demás, y como esta suma se hace infinitamente

19
20 TEMA 2. SIMETRÍA

pequeña cuando alguno de los ángulos se aproxima a 180◦ , es razonable que


será máxima cuando los tres ángulos sean iguales a 60◦ , algo que aunque
habrá que probar, apunta en una dirección para llegar a la solución.

2.2. Problemas geométricos


Es útil considerar la simetrı́a cuando tengamos ante nosotros un problema
geométrico. Por ejemplo,

Problema 19. Si P es un punto en el interior de un ángulo, buscar los


puntos A y B sobre los lados del ángulo que hacen que el triángulo P AB
tenga perı́metro mı́nimo.
Solución. Sean Q y R los puntos simétricos del punto P respecto de los
lados del ángulo, y A y B puntos buscados estos lados.

B
P

Q
Figura 4

Es evidente que AQ = AP y BR = BP , por lo que el perı́metro de


P AB será P A + AB + BP = QA + AB + BR, y resulta claro que esta suma
será minima cuando A y B estén sobre la recta QR.
Basándonos en el problema que acabamos de resolver podemos intentar
este otro:

Problema 20. Inscribir en un triángulo ABC un triángulo DEF con perı́-


metro mı́nimo.
Solución. Para cada punto D sobre la recta BC hacemos la construcción
del problema anterior, hallando ası́ los puntos E y F sobre las rectas AC y
2.3. PROBLEMAS NO GEOMÉTRICOS 21

AB tales que DEF tiene perı́metro mı́nimo para este D. Lo que tenemos que
hacer ahora es hallar para qué punto D sobre BC se obtiene el perı́metro
mı́nimo de estos triángulos.

R
C

E
D

A F B

Q
Figura 5

La solución está de nuevo en la simetrı́a. Sean Q y R los puntos simétricos


de D respecto de las rectas AB y AC respectivamente. Por ser AQ = AD =
AR, el triángulo AQR es isósceles y además ∠QAR = 2∠BAC. Observemos
que la base de este triángulo isósceles es el área del triángulo DEF . De
todos los triángulos isósceles con el ángulo desigual constante, el de menor
perı́metro será el que tenga menores los lados iguales AP = AQ, es decir
tenemos que elegir el punto D de manera que la distancia AD sea mı́nima.
Esto sucederá cuando AD sea una altura.
Repitiendo el razonamiento para los otros vértices resultarı́a igualmente
que BE y CF son alturas del triángulo ABC, es decir D, E y F son los pies
de las alturas del triángulo ABC, o DEF es el llamado triángulo órtico del
triángulo ABC.

2.3. Problemas no geométricos


Aunque es más fácil pensar en la simetrı́a cuando se trata de resolver un
problema geométrico, hay muchas otras ocasiones en las que la simetrı́a es
un gran recurso, y a veces el único.
22 TEMA 2. SIMETRÍA

Problema 21. Usar la simetrı́a para efectuar el producto

p = (x2 y + y 2 x + z 2 y)(xy 2 + yz 2 + zx2 ).

Solución. Podemos expresar, teniendo en cuenta la simetrı́a de la expresión


∑ ∑
en cada paréntesis, p = ( x2 y) ( xy 2 ) y entonces podemos calcular p sin
hacer los nueve productos resultantes de multiplicar cada término del primer
paréntesis por cada uno del segundo paréntesis:
(∑ )( )
p= x2 y xy 2 + yz 2 + zx2
∑ ∑ ∑
= x3 y 3 + x2 y 2 z 2 + x4 yz
( ) ( )
= x3 y 3 + y 3 z 3 + z 3 x3 + 3x2 y 2 z 2 + x4 yz + y 4 zx + z 4 xy .

Veamos ahora un problema aparecido en la Putnam Mathematical Com-


petition de 1980:
∫ π/2
dx
Problema 22. Calcular la integral √ .
0 1 + (tan x) 2
Solución. La función
1
f (x) = √
1 + (tan x) 2
del integrando no tiene una primitiva que pueda expresarse mediante funcio-
nes elementales.
Por ello, va a ser conveniente recurrir a una figura. En efecto, si tenemos
en cuenta que la función es simétrica respecto del punto ( π4 , 12 ), podremos
hallar la integral como un área, la mitad del rectángulo de base π2 y altura 1.

1−
2

π
− π

4 2

La simetrı́a central cuyo centro es el punto ( π4 , 21 ) viene dada por las


ecuaciones (x′ , y ′ ) = ( π2 −x, 1−y), por lo que para demostrar que la gráfica de
2.3. PROBLEMAS NO GEOMÉTRICOS 23
( )
y = f (x) en [0, π2 ] es simétrica bastará comprobar que f π2 − x + f (x) = 1.
En efecto,
(π ) 1 1
f − x + f (x) = √ ( )+ √
2 1 + tan 2 π
2
−x 1 + tan 2 x
1 1
= √ + √
2 2
1 + cot

x 1 + tan x
2
tan x 1
= √ + √
1 + tan x 1 + tan 2 x
2

=1.
Este problema es de la quinta Olimpiada Internacional:
Problema 23. Encontrar todas las soluciones x1 , x2 , x3 , x4 , x5 del sistema


 x5 + x2 = yx1




 x1 + x3 = yx2
x2 + x4 = yx3 .


 x3 + x5 = yx4



 x + x = yx
4 1 5

Solución. Si llamamos S = x1 + x2 + x3 + x4 + x5 , sumando todas las


ecuaciones tenemos 2S = yS. En el caso y = 2, si sumamos las tres ecuaciones
centrales tenemos x1 + x2 + 2x3 + x4 + x5 = 2(x2 + x3 + x4 ), de donde
x1 +x5 = x2 +x4 o x1 −x2 = x4 −x5 . Usando la simetrı́a, todas las diferencias
xi − xi+1 (considerando x6 = x1 ) son iguales, es decir, si y = 2, hay infinitas
soluciones de la forma x1 = x2 = x3 = x4 = x5 = λ. Ahora, si y ̸= 2, tenemos
S = 0, y sumando las tres ecuaciones centrales tenemos x3 = y(x2 +x3 +x4 ) =
y(x3 + yx3 ) = x3 (y + y 2 ), es decir, (y 2 + y − 1)x3 = 0. Si es y 2 + y − 1 ̸= 0 debe
ser x3 = 0, y lo mismo para las otras incógnitas. Si y 2 + y − 1 = 0, despejando
respectivamente de las ecuaciones primera, quinta y tercera, tenemos
x2 = yx1 − x5 ,
x4 = yx5 − x1 ,
x2 + x4 (yx1 − x5 ) − (yx5 − x1 ) (x1 + x5 )(y − 1)
x3 = = = = −y(x1 + x5 ).
y y y
Estos valores cumplen las otras dos ecuaciones, por ejemplo la segunda:
x1 + x3 = x1 − y(x1 + x5 ) = (1 − y)x1 − yx5 = y 2 x1 − yx5 = y(yx1 − x5 ) = yx2 ,
y la cuarta se deduce por la simetrı́a.
24 TEMA 2. SIMETRÍA

2.4. Problemas de la edición anterior


Problema 24. Demostrar que si al producto de cuatro números que están
en progresión aritmética le agregamos la cuarta potencia de la diferencia de
esta progresión obtenemos siempre un cuadrado perfecto. (Usar la simetrı́a
en la notación para facilitar la resolución.)
Solución. En lugar de escribir los términos de la sucesión como a, a + d,
a + 2d y a + 3d, llamamos m a la media aritmética de los dos extremos y los
números se pueden expresar en la forma
3d d d 3d
m− ,m − ,m + ,m +
2 2 2 2
siendo su producto
( )( )
9d2 d2 10d2 m2 9d4
m −
2
m −
2
= m4 − + .
4 4 4 16
Al añadir d4 obtenemos
( )2
10d2 m2 25d4 5d2
m −
4
+ = m −
2
.
4 16 4
Problema 25. Dadas las rectas l1 , l2 y l3 , que se cortan en un punto, junto
con un punto A sobre una de ellas construir un triángulo ABC que tenga a
l1 , l2 y l3 por bisectrices.
Solución. Supongamos que el punto dado A está sobre la recta l1 . Halla-
mos el punto M , simétrico de A respecto de l2 y el punto N , simétrico de M
respecto de l3 . Si llamamos C = AN ∩ l3 y B = CM ∩ l2 , el triángulo ABC
será el triángulo buscado.
Problema 26. Hacer uso del principio de razón insuficiente para minimizar
x21 + · · · + x2n cumpliendo las condiciones 0 6 xi 6 1 y x1 + · · · + xn = 1.
Demostrar la conjetura (para ello, considerar xi = n1 + ei ).
Solución. Por el principio de razón insuficiente podemos pensar que todos
los xi deben ser iguales, por lo que conjeturamos que xi = 1/n para todos
= 1, . . . , n. Para demostrarlo, sean expresemos xi = n1 + ei , donde
los i ∑
será ei = 0, y tendremos que
∑ ∑(1 )2 ∑ (
1 2ei
)
2 2
xi = + ei = + ei +
n n2 n
1 ∑ 2 ∑ 1 ∑ 2 1
= + e2i + ei = + ei > ,
n n n n
2.4. PROBLEMAS DE LA EDICIÓN ANTERIOR 25

cumpliéndose la igualdad cuando todos los ei son cero, es decir, cuando todos
los xi son iguales.
∫ 4 √
ln(9 − x)
Problema 27. Calcular la integral √ √ dx.
2 ln(9 − x) + ln(x + 3)
Solución. Llamando I a la integral
∫ 1que tenemos que calcular y haciendo
el cambio x = 3 + t, resulta que I = −1 f (t)dt siendo

ln(6 − t)
f (t) = √ √ .
ln(6 − t) + ln(6 + t)

Por ser f (1 − t) = 1 − f (t), la gráfica de f es simétrica respecto del punto


(0, 21 ). Teniendo en cuenta que
√ √
ln 7 ln 5
f (−1) = √ √ >√ √ = f (1) > 0,
ln 5 + ln 7 ln 5 + ln 7
podemos hallar la integral como la de un rectángulo con base 2 y altura

f (−1) − f (1) f (−1) + f (1) 1


f (1) + = = ,
2 2 2
de donde resulta que I = 1.

Problema 28. Factorizar la expresión b2 c3 − b3 c2 + c2 a3 − c3 a2 + a2 b3 − a3 b2 .


Solución. Como la expresión f (a, b, c) = b2 c3 −b3 c2 +c2 a3 −c3 a2 +a2 b3 −a3 b2
se anula cuando dos de las variables son iguales, entonces f (a, b, c) debe ser
divisible por (b − c)(c − a)(a − b). El cociente, que debe ser de grado 2 y
simétrico respecto de a, b y c. Considerando f (a, b, c) como polinomio en a el
coeficiente de a3 es c2 −b2 , y el coeficiente de a2 en el divisor (b−c)(c−a)(a−b)
es c − b, por lo que el coeficiente de a en el cociente debe ser b + c. La simetrı́a
respecto de las variables nos dice que

f (a, b, c)
= ab + bc + ca,
(b − c)(c − a)(a − b)

es decir,
f (a, b, c) = (b − c)(c − a)(a − b)(ab + bc + ca).
26 TEMA 2. SIMETRÍA

2.5. Problemas propuestos

2.1. Calcular la siguiente suma:


cos 1◦ + cos 2◦ + · · · + cos 178◦ + cos 179◦

2.2. Cualquier polinomio que sea simétrico en las variables x, y, z se puede


expresar en términos de los polinomios simétricos elementales σ1 = x + y + z,
σ2 = xy + yz + zx y σ3 = xyz. Ası́, por ejemplo, es x2 + y 2 + z 2 = σ12 − 2σ2 .
Expresar x3 + y 3 + z 3 en términos de los polinomios simétricos elementales.
2.3. Aplicar el principio de razón insuficente para intuir la solución del
sistema  3
 y − 9x + 27x − 27 = 0
 2

z 3 − 9y 2 + 27y − 27 = 0 .

 3
x − 9z 2 + 27z − 27 = 0
Resolver dicho sistema.
2.4. Construir un triángulo conociendo, en posición, sus tres bisectrices y
un punto sobre el perı́metro del triángulo.
2.5. En un plano se dan una recta y un punto no perteneciente a ella. Hallar
el lugar geométrico del tercer vértice de un triángulo equilátero, uno de cuyos
vértices es el punto dado y otro está sobre la recta.
Tema 3
Desigualdades

3.1. Medias
Dados n números no negativos x1 , x2 , · · · , xn vamos a considerar tres
formas de conseguir un valor promedio de todos ellos:

n
La media armónica: H = . (Suponemos en este caso
+ + ··· +
1
x1
1
x2
1
xn
que los números son todos positivos).

La media geométrica: G = n
x1 x2 · · · xn .

x1 + x2 + · · · + xn
La media aritmética: A = .
n

Ejemplo. Consideremos n = 3 y los números x = 1, y = 2, z = 4. Es


fácil comprobar que H = 12 7
, G = 2, A = 73 y que entonces, en este caso,
se cumple que H 6 G 6 A. En los apartados siguientes, veremos que esta
relación se cumple siempre y que todas las medias son iguales si y solo si
todos los números son iguales.

3.2. La desigualdad A-G


Comenzamos con una demostración elemental de la desigualdad A > G
y de que la igualdad A = G se cumple si y solo si x1 = x2 = · · · = xn .

3.2.1. Demostración por inducción

La demostración consta de dos partes:

27
28 TEMA 3. DESIGUALDADES

1. La desigualdad es cierta si n es un entero de la forma 2k .

Para k = 1, es decir, para n = 2, la desigualdad se convierte en

x+y √ √ √ √
> xy ⇔ x + y − 2 xy > 0 ⇔ ( x − y)2 > 0.
2

que es cierta y que se convierte en igualdad si y solo si x = y.

Supongamos que la desigualdad es cierta para n = 2k números y de-


mostremos que también lo es para 2n = 2k+1 números x1 , x2 , · · · , x2n .
Para ello consideramos los números

x1 + x2 x3 + x4 x2n−1 + x2n
y1 = , y2 = , · · · , yn = .
2 2 2

Sabemos que yi > x2i−1 x2i con igualdad si y solo si x2i−1 = x2i .
Entonces:

x1 + x2 + · · · + x2n y1 + y2 + · · · + yn
=
2n n
(1) √
> n y1 y2 · · · yn
(2) √
> 2n x1 x2 · · · x2n .

por lo que hemos demostrado la desigualdad. Para que (1) se convierta


en igualdad, todos los yi deben ser iguales y para que (2) también sea
igualdad, x2i−1 y x2i deben siempre coincidir. Por tanto, los xi deben
ser todos iguales.

2. Si la desigualdad es cierta para un cierto número n, también lo es para


n − 1.

Para ello, sean x1 , . . . , xn−1 números no negativos y sean

x1 + · · · + xn−1
y1 = x 1 , ..., yn−1 = xn−1 , yn = .
n−1
3.2. LA DESIGUALDAD A-G 29

Entonces,

x1 + · · · + xn−1 + x1 +···+x n−1
x1 + · · · + xn−1
n−1
> n x1 · · · xn−1
n n−1

x1 + · · · + xn−1 x1 + · · · + xn−1
> n x1 · · · xn−1
n−1 n−1
( )1− 1
x1 + · · · + xn−1 n 1
> (x1 · · · xn−1 ) n
n−1
( )n−1
x1 + · · · + xn−1
>x1 · · · xn−1
n−1
x1 + · · · + xn−1 √
> n−1 x1 · · · xn−1 .
n−1

3.2.2. Demostración por sustituciones


Vamos a demostrar la desigualdad
x1 + x2 + · · · + xn √
> n x1 x2 · · · xn
n
mediante sustituciones de los números xi por otros de manera que la
parte izquierda no varı́a y la derecha va aumentando. Al final, los xi
tendrán todos el mismo valor y la parte izquierda será igual a la parte
derecha. De aquı́ se concluirá la validez de la desigualdad.
Si los xi no son ya todos iguales a su media aritmética, que por conve-
niencia llamamos a, deben existir dos ı́ndices i y j, tale que xi < a < xj
(todos los números no pueden a la vez ser ni mayores ni menores que
su media aritmética). Entonces sustituimos los números xi y xj por
xi ′ = a y xj ′ = xi + xj − a. De esta manera, la suma total no cambia y
el producto aumenta. En efecto,

xi ′ xj ′ = a(xi + xj − a) = xi xj + (xj − a)(a − xi ) > xi xj ,

justificándose la desigualdad por la relación xi < a < xj . Después de


repetir el proceso, conseguiremos una igualdad a base de incrementar
el lado derecho y mantener igual el izquierdo.

3.2.3. Problemas resueltos


Veamos algunos ejemplos.
30 TEMA 3. DESIGUALDADES

Problema 29. Demostrar que si a, b y c son positivos, entonces

(b + c − a)(a + c − b)(a + b − c) 6 abc.

Solución. Llamando x = b + c − a, y = a + c − b y z = a + b − c tenemos

x+y y+z z+x


= c, = a, = b.
2 2 2

Entonces,
y+z z+x x+y
abc = · · >
2 2 2
√ √ √ √ √ √
> y z · z x · x y = xyz =
=(b + c − a)(a + c − b)(a + b − c).

Problema 30. Si a,b,c son tres reales positivos cualesquiera, demostrar


que:
a+b b+c c+a
+ + > 8.
c a b
Solución. Usamos la desigualdad entre las medias aritmética y geomé-
trica.
( )( )
a+b b+c c+a a b b c (c a)
+ + = + + + >
c a b c c a a b b
√ √ √
a b b c c a
=2 · ·2 · ·2 · = 8.
c c a a b b

3.3. Desigualdad entre las medias geométrica


y armónica
Aplicando la desigualdad A − G a los números 1
x1
, ..., x1n , obtenemos:

1
+ ··· + 1
1 1 n √
x1 xn
> n
... ⇒ 6 n
x1 . . . xn ⇒ H 6 G.
n x1 xn 1
x1
+ ··· + 1
xn

Nota: Según lo visto en 1.2, tenemos que H 6 G 6 A. Es evidente que la


igualdad se cumple si y solo si los números x1 , x2 , · · · , xn son todos iguales.
3.4. LA DESIGUALDAD DE CAUCHY-SCHWARZ 31

3.4. La desigualdad de Cauchy-Schwarz


3.4.1. La desigualdad de Cauchy-Schwarz
Bunyakovskii (1804-1889) publicó esta desigualdad en una monografı́a so-
bre desigualdades entre integrales en 1859, veinticinco años antes que Schwarz
(1843-1921), pero es más conocida como desigualdad de Cauchy-Schwarz.
La desigualdad de Cauchy-Schwarz nos dice que si x1 , . . . , xn e y1 , . . . , yn
son números reales cualesquiera se cumple:

(x1 y1 + · · · xn yn )2 6 (x21 + · · · + x2n )(y12 + · · · + yn2 ).

Para demostrarla, usaremos que si ax2 + bx + c > 0 para cualquier x, el


discriminante ∆ = b2 − 4ac no puede ser positivo.
Para ello, tengamos en cuenta que para cualquier λ ∈ R se cumple que

(λx1 + y1 )2 + · · · + (λxn + yn )2 > 0.

Desarrollando los cuadrados,

0 6 (λx1 + y1 )2 + · · · + (λxn + yn )2 =
= (x21 + · · · + x2n )λ2 + 2(x1 y1 + · · · xn yn )λ + (y12 + · · · + yn2 )

de donde

4(x1 y1 + · · · xn yn )2 − 4(x21 + · · · + x2n )(y12 + · · · + yn2 ) 6 0,

apareciendo la desigualdad buscada.


Además, vemos que la igualdad se cumplirá si y solo si los números si
x1 , . . . , xn e y1 , . . . , yn son proporcionales.

3.4.2. Problemas resueltos


Veamos algunas aplicaciones de la desigualdad de Cauchy-Schwarz.
Problema 31. Sean a, b, c, d, e números reales tales que a+b+c+d+e = 8
y a2 + b2 + c2 + d2 + e2 = 16. Hallar el valor máximo de e. Solución. Por la
desigualdad de Cauchy-Schwarz,

(a + b + c + d)2 6 (12 + 12 + 12 + 12 )(a2 + b2 + c2 + d2 ).

Entonces (8 − e)2 6 4(16 − e2 ), que desarrollando y simplificando nos da


e(5e − 16) 6 0. Esto nos dice que 0 6 e 6 16 5
y por tanto el valor máximo
16 6
pedido es 5 , que ocurre si a = b = c = d = 5 .
32 TEMA 3. DESIGUALDADES

Problema 32. Sean x, y, z > − 41 tales que x + y + z = 1. Calcular el


√ √ √
máximo valor de 4x + 1 + 4y + 1 + 4z + 1. Solución. Por la desigualdad
de Cauchy-Schwarz,
√ √ √
4x + 1 + 4y + 1 + 4z + 1 =
√ √ √
= 1 · 4x + 1 + 1 · 4y + 1 + 1 · 4z + 1 6
√ √
6 12 + 12 + 12 (4x + 1) + (4y + 1) + (4z + 1) =
√ √ √
= 3 4(x + y + z) + 3 = 21.

Problema 33. Demostrar que un triángulo de lados x, y, z es semejante a


un triángulo de lados a, b, c si y solo si
√ √ √ √
ax + by + cz = (a + b + c)(x + y + z).

Solución. Usando la desigualdad de Cauchy-Schwarz con las raı́ces cuadradas


de los lados, obtenemos que
√ √ √ √
ax + by + cz 6 (a + b + c)(x + y + z),

cumpliéndose la igualdad si y solo si los números x, y, z son proporcionales a


a, b, c, y esa es la condición que determina a dos triángulos semejantes.
Problema 34. Siendo a y b números reales, hallar el valor máximo y mı́nimo
de la expresión a cos θ + b sen θ. Solución. Usando la desigualdad de Cauchy-
Schwarz,

(a cos θ + b sen θ)2 6 (a2 + b2 )(cos2 θ + sen2 θ) = a2 + b2 .

La igualdad se cumple cuando a sen θ = b sen θ, es decir cuando tgθ = ab .


Habrá dos valores entre 0 y 2π que consigan este valor.
√ Uno corresponderá
√ al
mı́nimo y otro al máximo, que serán entonces − a2 + b2 y a2 + b2 , res-
pectivamente. Estas fórmulas sirven para el caso a = 0, pues entonces la
expresión se reduce a b sen θ.

3.5. La desigualdad de Chebyshev


La desigualdad de Chebyshev se aplica a sucesiones monótonas en la
misma dirección. Para concretar, supongamos que x1 , . . . , xn e y1 , . . . , yn son
tales que x1 6 · · · 6 xn y y1 6 · · · 6 yn . Entonces se cumple que
(x1 + · · · + xn )(y1 + · · · + yn )
x1 y1 + · · · + xn yn > .
n
3.5. LA DESIGUALDAD DE CHEBYSHEV 33

Demostremos aquı́ la desigualdad par n = 3, siendo la demostración similar


para cualquier n.
Sean a, b, c, x, y, z números reales tales que a 6 b 6 c y x 6 y 6 z.
Queremos demostrar que 3(ax + by + cz) > (a + b + c)(x + y + z). Para ello,
restamos el segundo miembro al primero y obtenemos

2ax + 2by + 2cz − bx − cx − ay − cy − az − bz,

que podemos expresar en la forma

ax − ay + ax − az + by − bx + by − bz + cz − cx + cz − cy,

que a su vez es el resultado de desarrollar

(a − b)(x − y) + (a − c)(x − z) + (b − c)(y − z).

Al ser ésta una expresión no negativa, queda probada la desigualdad.


Como ejemplo de la desigualdad de Chebyshev, resolvamos el siguiente
problema:

Problema 35. Siendo a, b, c y d números positivos, demostrar que

16(a3 + b3 + c3 + d3 ) > (a + b + c + d)3 .

Solución. Sin pérdida de generalidad suponemos que a 6 b 6 c 6 d. Apli-


cando la desigualdad de Chebyshev,

(a + b + c + d)(a + b + c + d) (a + b + c + d)2
a2 + b2 + c2 + d2 > = .
4 4

Como todos los números son positivos, tendremos que a2 6 b2 6 c2 6 d2 .


Aplicando de nuevo la desigualdad de Chebyshev,

(a2 + b2 + c2 + d2 )(a + b + c + d)
a3 + b3 + c3 + d3 > >
4
1
(a + b + c + d)2 (a + b + c + d)
> 4
=
4
1
= 16 (a + b + c + d)2 .

El siguiente problema también puede resolverse usando la desigualdad de


Chebyshev:
34 TEMA 3. DESIGUALDADES

Problema 36. Siendo x, y, z números positivos, hallar el valor mı́nimo de


la expresión
x y z
+ + .
y+z z+x x+y
Solución. Podemos suponer, sin pérdida de generalidad que x 6 y 6 z.
Entonces será y + z > z + x > x + y y también y+z1
6 z+x
1
6 1
x+y
. Aplicando
la desigualdad de Chebyshev,
( )
1 1 1
x y z (x + y + z) y+z
+ z+x + x+y
+ + > .
y+z z+x x+y 3
Por otro lado, usando la desigualdad entre las medias armónica y aritmé-
tica,
3 (x + y) + (y + z) + (z + x) 2(x + y + z)
1 1 1 > = .
y+z
+ z+x
+ x+y
3 3
De aquı́ deducimos que
( ) 9 x y z 3
1
(x + y + z) y+z 1
+ z+x + 1
x+y
> ⇒ + + > .
2 y+z z+x x+y 2
Como para x = y = z obtenemos la igualdad, el valor obtenido no puede
mejorarse.

3.6. Problemas de la edición anterior


Problema 37. Demostrar que si a + b + c > 3, entonces a2 + b2 + c2 > 3.
Solución. Usamos la desigualdad de Cauchy-Schwarz:
9 6 (a + b + c)2 = (a · 1 + b · 1 + c · 1)2 6 (12 + 12 + 12 ) · (a2 + b2 + c2 ),
de donde a2 + b2 + c2 > 3.
Problema 38. Demostrar que si a, b y c son positivos, entonces
(a2 b + b2 c + c2 a)(ab2 + bc2 + ca2 ) > 9a2 b2 c2

Solución. Dividiendo por abc, la desigualdad propuesta es equivalente a


( )( )
a b c b c a
+ + + + > 9.
c a b c a b
Aplicando la desigualdad A − G a cada paréntesis,
( )( ) √ √
a b c b c a 3 a b c 3 b c a
+ + + + >3· · · ·3· · · = 9.
c a b c a b c a b c a b
3.6. PROBLEMAS DE LA EDICIÓN ANTERIOR 35

Problema 39. Sean a, b, c, d > 0. Demostrar que


1 1 4 16 64
+ + + > .
a b c d a+b+c+d

Solución. Aplicamos la desigualdad H ≤ A a los números a y b:


2 a+b 1 1 4
1 1 6 ⇒ + > .
a
+ b
2 a b a+b

Repitiendo el proceso dos veces,


4 4 4 16
+ >4· = ,
a+b c (a + b) + c a+b+c
16 16 4 64
+ > 16 · = .
a+b+c d (a + b + c) + d a+b+c+d

Problema 40. Supongamos que a + b + c = 1, siendo a > 0, b > 0, c > 0.


Demostrar que √ √ √
9a + 1 + 9b + 1 + 9c + 1 6 6.
√ √ √
Solución. Consideramos las ternas ( 9a + 1, 9b + 1, 9c + 1) y (1, 1, 1).
Entonces, aplicando la desigualdad de Cauchy-Schwarz,
√ √ √ √ √
9a + 1 + 9b + 1 + 9c + 1 6 1 + 1 + 1 9a + 1 + 9b + 1 + 9c + 1 =
√ √
= 3 9(a + b + c) + 3 = 6.

Problema 41. Sean a, b, c números positivos. Demostrar que


a+b+c 1 1 1
6 2 + 2 + 2.
abc a b c
Aplicamos la desigualdad de Cauchy-Schwarz a las ternas ( 1b , 1c , a1 ) y
( 1c , a1 , 1b ).
Entonces,
√ √
1 1 1 1 1 1 1 1 1 1 1 1
+ + 6 2
+ 2+ 2· 2
+ 2 + 2 = 2 + 2 + 2.
bc ca ab b c a c a b a b c
36 TEMA 3. DESIGUALDADES

3.7. Problemas propuestos

3.1. Demostrar que si m y n son enteros positivos, entonces 2 siempre está en-
m 10n+m
tre los números n
y m+4n
.
3.2. Sobre un punto arbitrario X del diámetro AB de una circunferencia,
levantar una perpendicular XY hasta un punto Y sobre la circunferencia. De-
mostrar que XY es la media geométrica de los segmentos AX y XB, y usarlo
para justificar geométricamente la desigualdad entre las medias aritmética y
geométrica.
3.3. Demostrar que si x, y, z son números positivos, entonces
√ √ √
x+y y+z z+x √
+ + 6 6.
x+y+z x+y+z x+y+z

3.4. Demostrar que si a, b, c son números positivos, entonces


1 b+c c+a b+a a+b+c
( + + )≥ √
3
.
2 a b c abc

3.5. Demostrar que si a, b, c son números positivos tales que a2 + b2 + c2 = 3,


entonces
1 1 1 3
+ + > .
1 + ab 1 + bc 1 + ca 2
Tema 4
Geometrı́a del triángulo

4.1. Notación en el triángulo


Es habitual llamar a un triángulo ABC, indicando A, B y C los vértices
o los ángulos de dicho triángulo.
La longitud de los lados BC, CA y AB se designa normalmente con las
letras a, b y c.
El área del triángulo ABC se representará con el sı́mbolo (ABC), o con
SABC .
R y r serán normalmente los radios de la circunferencia circunscrita e
inscrita al triángulo.
El baricentro, circuncentro, ortocentro e incentro se representarán con las
letras G, O, H e I, respectivamente.

4.2. Teoremas de los senos y del coseno


El teorema del coseno dice que en cualquier triángulo ABC se cumple
que c2 = a2 + b2 − 2ab cos C (ası́ como las otras dos igualdades que resultan
de permutar las letras de ésta).
El teorema de los senos (generalizado) nos dice que en un triángulo ABC

a b c
= = = 2R.
sen A sen B sen C

Para demostrarlo tenemos en cuenta que los ángulos inscritos que abarcan
el mismo arco son iguales, y que dos ángulos opuestos de un cuadrilátero
inscrito suman 180◦ (y por tanto tienen el mismo seno).

37
38 TEMA 4. GEOMETRÍA DEL TRIÁNGULO

D D
O O

B a C B a C
A
Figura 6

En efecto, basta trazar el diámetro CD y obtenemos que


a a
= = CD = 2R.
sen A sen D

4.3. Área de un triángulo


La forma más corriente de hallar el área de un triángulo es usar una
altura. Sin embargo, hay muchas otras formas y algunas de ellas son muy
útiles en la resolución de problemas.

c b
h

B a C
Figura 7

Por ejemplo, si h es la altura trazada por el vértice A tenemos

ah ab sen C abc
(ABC) = = = .
2 2 4R
Por otro lado, si llamamos s al semiperı́metro del triángulo, es decir si
s = a+b+c
2
, podemos dividir el triángulo ABC en tres triángulos con vértice
común el incentro y obtener

(ABC) =(AIB) + (BIC) + (CIA) =


cr ar br a+b+c
= + + = r = sr.
2 2 2 2
4.4. EL TEOREMA DE LA BISECTRIZ 39

r Ir

r
B C
Figura 8

Por último, citemos la fórmula de Herón:



(ABC) = s(s − a)(s − b)(s − c).

Esta fórmula se obtiene a partir del teorema del coseno. En efecto,

a2 b2 a2 b2 ( )
(ABC)2 = sen2 C = 1 − cos2 C =
4 4
2 2
ab
= (1 + cos C) (1 − cos C) =
4 ( )( )
a2 b2 a2 + b2 − c2 a2 + b2 − c2
= 1+ 1− =
4 2ab 2ab
1 ( )( )
= 2ab + a2 + b2 − c2 2ab − a2 − b2 + c2 =
16
1 ( )( )
= (a + b) − c2 c2 − (a − b)2 =
16
a+b+c a+b−c c+a−b c−a+b
= · · · =
2 2 2 2
=s(s − c)(s − b)(s − a).

A partir de la fórmula de Herón podemos obtener unas fórmulas explı́citas


para calcular R y r:

abc (s − a)(s − b)(s − c)
R= √ , r= .
4 s(s − a)(s − b)(s − c) s

4.4. El teorema de la bisectriz


Las bisectrices de un triángulo cumplen muchas propiedades, que es con-
veniente conocer cuando nos encontramos algún problema en el que aparecen.
40 TEMA 4. GEOMETRÍA DEL TRIÁNGULO

B L C
Figura 9

Teorema de la bisectriz. Si la bisectriz interior del ángulo A del triángulo


ABC corta al lado BC en L, entonces

BL AB
= .
LC AC
Demostración. En efecto, tenemos (ABL)/(ALC) = BL/LC, ya que al tener
ambos triángulos la misma altura (la distancia de A a la recta BC), las áreas
son proporcionales a las bases. Pero también, considerando como bases de
estos triángulos los lados AB y AC, respectivamente, y teniendo en cuenta
que el punto L equidista de las rectas AB y AC, tendremos (ABL)/(ALC) =
AB/AC, lo que demuestra el teorema de la bisectriz.
Análogamente, para la bisectriz exterior, tenemos (ver la observación so-
bre los signos en la pág 45):

Problema 42. Si la bisectriz exterior del ángulo A corta en L a la prolon-


gación del lado BC, entonces

BL AB
=− .
LC AC

B C L
Figura 10

4.5. El teorema de Stewart


Se llama ceviana a un segmento que une un vértice de un triángulo con un
punto cualquiera del lado opuesto. Por ejemplo, en la figura 11, el segmento
AP es una ceviana.
4.5. EL TEOREMA DE STEWART 41

c b
p
m n
B P C
Figura 11

El teorema de Stewart (Matthew Stewart, 1717-1785) nos dice lo que


mide una ceviana.
Teorema de Stewart. Si el punto P está sobre el lado BC de manera que
BP = m y P C = n, entonces la longitud p de la ceviana AP cumple

b2 m + c2 n = a(p2 + mn).

Solución. Damos una demostración de este teorema basada en el teorema


del coseno. En efecto, como cos ∠AP B + cos ∠AP C = 0,
m2 + p2 − c2 n2 + p2 − b2
+ =0⇒
2mp 2np
⇒n(m2 + p2 − c2 ) + m(n2 + p2 − b2 ) = 0 ⇒
⇒b2 m + c2 n = (m + n)p2 + nm2 + mn2 = (m + n)(p2 + mn) ⇒
⇒b2 m + c2 n = a(p2 + mn).

Problema 43. Hallar la longitud ma de la mediana trazada desde A.


Solución. En este caso tenemos m = n = a2 , por lo que según el teorema
de Stewart,
( )
2a 2a a2 a2
b + c = a ma + 2
⇒ b + c = 2ma +
2 2 2

2 2 4 2

2b2 + 2c2 − a2 2b2 + 2c2 − a2 1√ 2
⇒ma =
2
⇒ ma = = 2b + 2c2 − a2 .
4 4 2
Problema 44. Hallar la longitud ta de la bisectriz interior del ángulo A.

c b
ta
m n
B P C
Figura 12
42 TEMA 4. GEOMETRÍA DEL TRIÁNGULO

Solución. Teniendo en cuenta el teorema de la bisectriz, podemos llamar


k = mc = nb .
De a = m + n = k(b + c) deducimos que

a ac ab
k= ⇒m= , n= .
b+c b+c b+c
Entonces, aplicando el teorema de Stewart,
( )
2 ca 2 ab 2 a2 bc
abc = b +c = a ta +
b+c b+c (b + c)2
√ ( )
a2 bc a2
⇒ bc = ta +
2
⇒ ta = bc 1 − .
(b + ac)2 (b + c)2

4.6. El teorema de Ceva


Sean X, Y , Z puntos de los lados BC, CA y AB respectivamente de un
triángulo ABC.

Z
Y
P

B X C
Figura 13

El teorema de Ceva (Giovanni Ceva, 1647-1734) afirma: Si las tres cevia-


nas AX, BY y CZ son concurrentes, entonces

BX CY AZ
· · = 1.
XC Y A ZB
Para demostrar este teorema usamos que las áreas de los triángulos con
alturas iguales son proporcionales a las bases de los triángulos. Supongamos
que las tres cevianas AX, BY y CZ se cortan en un punto P . Entonces

BX (ABX) (P BX) (ABX) − (P BX) (ABP )


= = = = .
XC (AXC) (P XC) (AXC) − (P XC) (CAP )
4.6. EL TEOREMA DE CEVA 43

Análogamente,
CY (BCP ) AZ (CAP )
= , = .
YA (ABP ) ZB (BCP )

Al multiplicar las tres fracciones obtenemos el resultado deseado.


El recı́proco del teorema de Ceva, que también es cierto, es muy útil:
BX CY AZ
Si · · = 1, entonces las cevianas son concurrentes.
XC Y A ZB
Para demostrarlo, usamos el método de la posición falsa. Suponemos que las
cevianas AX y BY se cortan en P y que la recta CP corta al lado AB en
Z ′ . Como según el teorema de Ceva se cumple que
BX CY AZ ′
· · = 1,
XC Y A Z ′ B
resulta que
AZ AZ ′ AB AZ AZ ′ AB
= ′ ⇒ = + 1 = ′ + 1 = ′ ⇒ Z = Z ′.
ZB ZB ZB ZB ZB ZB
Problema 45. Usar el teorema de Ceva para demostrar que
1. Las medianas de un triángulo son concurrentes.

2. Las bisectrices interiores de un triángulo son concurrentes.

3. Las alturas de un triángulo son concurrentes.

Solución: En el caso de las medianas BX = XC, CY = Y A y AZ = ZB,


por lo que
BX CY AZ
· · = 1 · 1 · 1 = 1,
XC Y A ZB
y, según el recı́proco del teorema de Ceva, las medianas son concurrentes.
En el caso de las bisectrices, podemos usar el teorema de la bisectriz para
obtener que
BX CY AZ AB BC AC
· · = · · = 1,
XC Y A ZB AC BA CB
resultando también cevianas concurrentes.
Por último, para las alturas tenemos que
BX CY AZ AB · cos A BC · cos C AC · cos A
· · = · · = 1.
XC Y A ZB AC · cos C BA · cos A BC · cos B
44 TEMA 4. GEOMETRÍA DEL TRIÁNGULO

Problema 46. Si X, Y , Z son los puntos de contacto de la circunferencia


inscrita en el triángulo ABC con los lados BC, CA y AB respectivamente,
demostrar que AX, BY y CZ son concurrentes. (El punto de intersección
se llama punto de Gergonne del triángulo ABC).
A
Y
Z G

B X C
Figura 14

Solución. Usaremos que las tangentes a una circunferencia desde un punto


exterior a la misma miden lo mismo. Entonces,
BX CY AZ AB CB AB
· · = · · = 1.
XC Y A ZB AC BA CB
Recordemos que una bisectriz exterior de un triángulo divide en dos el
ángulo formado por un lado y la prolongación de otro.

4.7. El teorema de Menelao


El teorema de Menelao (Menelao de Alejandrı́a, sobre 70-130 d.C.) pro-
porciona un criterio de alineación, lo mismo que el teorema de Ceva propor-
ciona un criterio de concurrencia.
Teorema de Menelao. Sean X, Y y Z puntos respectivamente sobre los
lados BC, AC y AB (o sus prolongaciones). Entonces, una condición nece-
saria y suficiente para que los puntos X, Y , Z estén alineados es que
BX CY AZ
· · = −1.
XC Y A ZB
A

B′ Z

A′ Y
C′

B C X
Figura 15
4.8. PROBLEMAS DE LA EDICIÓN ANTERIOR 45

Sobre los signos. En la llamada Geometrı́a Moderna se usan distancias


y áreas con signo. Ası́ tenemos, por ejemplo, AX = −XA y (ABC) =
−(ACB). También, las perpendiculares trazadas sobre una recta tendrán
longitudes con distinto signo, si se han trazado desde lados opuestos de la
recta. Por ejemplo, en la figura 13 las longitudes BX y XC tienen signo
distinto ya que para ir de BX y XC tienen sentidos opuestos. También, en
esta figura, A y C quedan a distinto lado de la recta XZ, por lo que si A′
y C ′ son las proyecciones de A y C, las distancias AA′ y CC ′ se consideran
con distinto signo.
Demostración del teorema de Menelao. Utilizando triángulos semejantes
tenemos
BX BX BB ′ CY CC ′ AZ AA′
=− =− , =− , =− ,
XC CX CC ′ YA AA′ ZB BB ′
de donde, multiplicando, obtenemos

BX CY AZ
· · = −1.
XC Y A ZB
El recı́proco del teorema de Menelao se demuestra de forma idéntica al
recı́proco del teorema de Ceva.

4.8. Problemas de la edición anterior


Problema 47. Demostrar que dos bisectrices exteriores de un triángulo y
la bisectriz interior del tercer ángulo son concurrentes. El punto común se
denomina excentro, porque es el centro de una circunferencia tangente a un
lado y a las prolongaciones de los otros dos.
Solución. Como muestra la figura, supongamos que las bisectrices exterio-
res a los ángulos A y C cortan a los las prolongaciones de los lados opuestos
en X y Z, respectivamente y que la bisectriz del ángulo B corta al lado AC
en Y . Entonces,

Z
A
P
Y

B C X
Figura 16
46 TEMA 4. GEOMETRÍA DEL TRIÁNGULO

Entonces, por los teoremas de las bisectrices interior y exterior,


( ) ( )
BX CY AZ AB BC AC
· · = − · · − = 1,
XC Y A ZB AC AB BC

por lo que AX, BY y CZ se cortan en un punto P .

Problema 48. Demostrar que las medianas AX, BY , CZ de un triángulo


dividen a éste en seis triángulos de la misma área. Usar esto para deducir
que, por ejemplo, AX = 3 · GX.

Solución. En el triángulo de la figura, las áreas con la misma letra son


iguales, ya que son triángulos con la misma base y la misma altura.

Z f e Y
G
f e
d d
B X C
Figura 17

Lo mismo les ocurre a los triángulos ABX y AXC, de donde 2f + d = 2e + d,


es decir f = d. De forma parecida, resulta d = e, y los seis triángulos tienen
la misma área.

Ahora, (AGB) = 2 · (GXB), y los dos triángulos tienen la misma altura,


obtenemos AG = 2 · GX, o AX = 3 · GX.

Problema 49. Supongamos que las bisectrices exterior e interior del ángulo
A del triángulo ABC cortan a la circunferencia circunscrita a ABC en los
puntos P y Q. Demostrar que P Q es el diámetro de dicha circunferencia
que divide a BC por la mitad. Demostrar también que si I es el incentro de
ABC, es QB = QI = QC.
Aquı́ usaremos que las dos bisectrices de un ángulo son perpendiculares.
Según esto, el ángulo QAP es recto, por lo que P Q es un diámetro de la
circunferencia circunscrita a ABC.
4.8. PROBLEMAS DE LA EDICIÓN ANTERIOR 47

Q
A

B C

P
Figura 18

Ahora, los ángulo ∠BQP y ∠P QC son iguales pues los arcos abarcados
por los mismos son iguales a los abarcados por cada una de las mitades del
ángulo A. Entonces, el triángulo BQC es isósceles y P Q es la mediatriz del
lado BC, cortando a éste en su punto medio.
Razonemos ahora que el triángulo P BI es isósceles. Llamemos α, β y γ
a las mitades de los ángulos A, B y C. Entonces ∠IBC = β y ∠CBP = α,
ası́ que ∠IBP = α + β. Por otro lado, también es ∠BIP = α + β, por ser
∠IBA = β y ∠BAI = α.
De forma parecida se razona que el triángulo P CI es isósceles, resultando
que P B = P I = P C.

Problema 50. La figura siguiente muestra que la circunferencia inscrita al


triángulo ABC toca a los lados BC, CA y AB de éste en los puntos X, Y y
Z, y que la circunferencia exinscrita con centro IB toca en Y ′ al lado AC y
en X ′ , Z ′ a las prolongaciones de BC y BA.
Demostrar que

1. BX = ZB = s − b, CY = XC = s − c, AZ = Y A = s − a.

2. BX ′ = BZ ′ = s.

3. AY ′ = s − c, CY ′ = s − a, Y Y ′ = c − a.

4. (ABC) = rA (s − a) = rB (s − b) = rC (s − c), siendo rA , rB , rC los radios


de las circunferencias exinscritas que son tangentes, respectivamente,
a los lados BC, CA y AB.
48 TEMA 4. GEOMETRÍA DEL TRIÁNGULO

Z′
A

Z Y′ IB
I
Y
B X C X′
Figura 19

Solución. Usando que las tangentes a una circunferencia trazadas desde


un punto miden lo mismo podemos llamar u = AZ = Y A, v = BX = ZB,
w = CY = XC.
Resolviendo el sistema

u + v = c, v + w = a, w + u = b

obtenemos u = s − a, v = s − b, w = s − c, que es lo que pide el primer


apartado.
Que BX ′ = BZ ′ se deduce de que ambas son tangentes desde el punto B
a la circunferencia exinscrita. Además,

BX ′ + BZ ′ =(BC + CX ′ ) + (BA + AZ ′ ) =
=(BC + BA) + (CY ′ + AY ′ ) = a + c + b = 2s,

es decir BX ′ = BZ ′ = s.
Ahora, AY ′ = AZ ′ = BZ ′ − BA = s − c, y CY ′ = CX ′ = BX ′ − BC =
s − a. Por último Y Y ′ = CY ′ − CY = (s − a) − (s − c) = c − a.
Demostremos ahora que (ABC) = rB (s − b).

(ABC) =(BZ ′ IB ) + (BX ′ IB ) − (Z ′ AIB ) − (AIB C) − (X ′ CIB ) =


= 21 srB + 12 srB − 12 (s − c)rB − 12 brB − 12 (s − a)rB =
= 12 (a − b + c)rB = 21 (s − b)rB .
4.9. PROBLEMAS PROPUESTOS 49

Problema 51. Si X, Y , Z son puntos sobre los lados BC, CA y AB, res-
pectivamente, del triángulo ABC de manera que X es el punto que divide el
perı́metro en dos partes iguales desde A, Y desde B y Z desde C, demostrar
que AX, BY y CZ son concurrentes. (El punto de intersección se llama
punto de Nagel del triángulo ABC).
Teniendo en cuenta que AB + BX = CX + CA = s, resulta que

Z Y
N

B X C
Figura 20

BX = s − AB = s − c y CX = s − CA = s − b. Las distancias CY , Y A,
AZ y ZB pueden obtenerse de forma similar, resultando que

BX CY AZ s−c s−a s−b


· · = · · = 1.
XC Y A ZB s−b s−c s−a
Aplicando el teorema de Ceva, las rectas AX, BY y CZ se cortan en un
punto.

4.9. Problemas propuestos

4.1. Demostrar la siguientes fórmulas del triángulo:


√ √ √
A (s − b)(s − c) A s(s − a) A (s − b)(s − c)
sen = , cos = , tan = .
2 bc 2 bc 2 s(s − a)

4.2. Supongamos que la bisectriz del ángulo A del triángulo ABC corta a
la recta BC en L. Sean I el incentro del triángulo ABC, e Ia el excentro
correspondiente al vértice A. Demostrar que:

AI LI
=−
AIa LIa
50 TEMA 4. GEOMETRÍA DEL TRIÁNGULO

4.3. Sean AD, BE, CF tres rectas cevianas concurrentes del triángulo ABC,
y supongamos que la circunferencia que pasa por D, E y F corta a los lados
BC, CA, AB también en D′ , E ′ , F ′ . Demostrar que AD′ , BE ′ , CF ′ son
concurrentes.
4.4. Supongamos que la bisectriz del ángulo A del triángulo ABC corta a
la recta BC en L demostrar que si se cumple la relación
1 1 2
+ = ,
BL2 LC 2 AL2
entonces el ángulo A es recto.
4.5. Demostrar que las tangentes a la circunferencia circunscrita de un
triángulo en los vértices del triángulo cortan a los lados opuestos del triángulo
en puntos alineados.
Tema 5
Coordenadas baricéntricas

5.1. Geometrı́a de masas


5.1.1. Introducción
El baricentro de un triángulo recibe ese nombre porque el sistema formado
por tres masas iguales situadas en los vértices es equivalente a otro sistema
en el que las tres masas estén situadas en ese punto. Para formalizar esto
definamos los puntos con masa y las operaciones entre ellos.

1. Un punto con masa es un par (u, A) formado por un número u, la masa,


y un punto A. Lo escribiremos en la forma uA.

2. Definimos uA + vB = (u + v)Z siendo Z el punto situado sobre la


recta AB tal que AZ : ZB = v : u. Si imaginamos una balanza con
extremos A y B y pesos u y v en esos puntos, la balanza se equilibrarı́a
si colocamos el fiel en el punto Z.)

5.1.2. Propiedades
Esta definición de puntos con masa y centro de masas que acabamos de
hacer tiene interesantes propiedades. Destacamos las siguientes:

1. (Clausura) La suma está unı́vocamente determinada.

2. (Conmutatividad) uA + vB = vB + uA (basta ver la balanza desde el


otro lado.)

3. (Asociatividad) uA + (vB + wC) = (uA + vB) + wC. Esta propiedad es


equivalente al teorema de Menelao. El punto resultante de cualquiera
de estas dos operaciones es el centro de masas del sistema formado por
uA, vB y wC.

51
52 TEMA 5. COORDENADAS BARICÉNTRICAS

5.1.3. Problemas resueltos


El concepto de centro de masas puede aplicarse para resolver un deter-
minado tipo de problemas.

Problema 52. En el △ABC sea D el punto medio de BC y E es el punto


de trisección del segmento AC más cercano a A. Sea G = BE ∩ AD. Hallar
AG : GD y BG : GE.

A(2)

E(3)
G(4)

B(1) D(2) C(1)


Figura 21

Solución. Consideramos la Figura 21. Asignamos peso 2 a A y 1 tanto a


B como a C. Entonces 2A + 1C = 3E y 1B + 1C = 2D. El centro de masas
del sistema es 2A + 1B + 1C = 1B + 3E = 2A + 2D = 4G. Entonces es
AG : GD = 2 : 2 = 1 : 1 y BG : GE = 3 : 1.

Problema 53. En el △ABC, D está sobre AB y E está sobre BD. Sea


F = AE ∩ CD. Sabiendo que AD = 3, DB = 2, BE = 3 y EC = 4 hallar
EF : F A.

A(8)

D(20) F(29)
2
3 4
B(12) E(21) C(9)
Figura 22

Solución. Asignamos masas 8, 12 y 9 a A, B y C. Entonces, 20D = 8A +


12B y 21E = 12B + 9C. El centro de masas del sistema es 29F = 21E + 8A,
por lo que resulta que EF : F A = 8 : 21.
5.2. COORDENADAS BARICÉNTRICAS 53

Problema 54. Demostrar que las medianas de un triángulo son concurrentes


y que el punto de concurrencia divide a cada mediana en la razón 2:1.
Solución. Sean D, E, F son los puntos medios de los lados BC, CA, AB
respectivamente. Entonces 2D = 1B + 1C, 2E = 1C + 1A, 2F = 1A + 1B.
El centro de masas del sistema será 3G = 1A + 1B + 1C = 1A + 2D =
1B + 2E = 1C + 2F . Por tanto, G debe estar en AD, BE y CF siendo
además AG : GD = BG : GE = CG : GF = 2 : 1.

5.2. Coordenadas baricéntricas


5.2.1. Coordenadas baricéntricas homogéneas
Las coordenadas baricéntricas son una herramienta muy potente para
hacer investigaciones sobre la geometrı́a del triángulo.
Si ABC es un triángulo, que llamaremos triángulo de referencia, a cual-
quier punto P de su plano le podemos asignar unos números u, v, w tales que
(u + v + w)P = uA + vB + wC, es decir tales que P es el centro de masas
del sistema formado por uA, vB y wC.
Esto permite definir a P como centro del masas del sistema formado
por los puntos A, B, C con las masas u, v, w. Esto permite llamar a u, v, w
las coordenadas baricéntricas del punto P respecto del triángulo ABC, y
por ser dichos números únicos salvo un factor de proporcionalidad, usamos
el sı́mbolo P = (u : v : w) para indicar que u, v, w son las coordenadas
baricéntricas del punto P .
Propiedad. Las coordenadas baricéntricas de P son proporcionales a las
áreas de los triángulos P BC, P CA, P AB.

B X C
Figura 23

Solución. En efecto, sea P = (u : v : w) y sea X sobre BC tal que


BX : XC = w : v. Entonces X es el centro de masas de vB y wC, y P es
el centro de masas de uA y (v + w)X. Entonces es P X : AP = u : (v + w)
54 TEMA 5. COORDENADAS BARICÉNTRICAS

y, por tanto, P X : AX = u : (u + v + w). Pero, los triángulos P BC y ABC


comparten la base BC, por lo que obtenemos la relación entre las áreas:
(P BC) : (ABC) = u : (u + v + w). Como podemos hacer lo mismo para las
otras coordenadas, resulta que

(P BC) : (P CA) : (P AB) = u : v : w.

5.2.2. Problemas resueltos


Problema 55. Hallar las coordenadas baricéntricas del baricentro, incentro
y circuncentro.
A A A

O
G I

B C
B C B C
Figura 24
Solución. El baricentro G tiene coordenadas baricéntricas homogéneas
(1 : 1 : 1), ya que las áreas GBC, GCA y GAB son iguales.
El incentro I tiene coordenadas baricéntricas homogéneas (a : b : c), ya
que si r es el radio de la circunferencia inscrita, las áreas de los triángulos
IBC, ICA e IAB son, respectivamente 12 ar, 12 br y 12 cr.
Las coordenadas del circuncentro O son
△OBC : △OCA : △OAB
= 12 R2 sen 2A : 12 R2 sen 2B : 12 R2 sen 2C
= sen A cos A : sen B cos B : sen C cos C
b2 + c2 − a2 c2 + a2 − b2 a2 + b2 − c2
=a · :b· :c·
2bc 2ac 2ab
a2 (b2 + c2 − a2 ) : b2 (c2 + a2 − b2 ) : c2 (a2 + b2 − c2 ).

Problema 56. Hallar las coordenadas de los excentros.


Solución. Consideramos la figura 20, en la que se muestran el triángulo
ABC, el excentro Ib , centro de la circunferencia exinscrita que toca al lado
AC y a las prolongaciones de BC y BA.
Las coordenadas baricéntricas de Ib son

△Ib BC : △Ib CA : △Ib AB = arb : −brb : crb = a : −b : c.


5.3. AREA DE UN TRIÁNGULO 55

A r

B C
Figura 25

Observemos que la orientación del triángulo Ib CA es distinta de la de los


otros dos, y por ello resulta signo negativo en la segunda coordenada. De
igual forma obtenemos que Ia = (−a : b : c) y que Ic = (a : b : −c).

5.3. Area de un triángulo


Si P = (x1 , y1 ), Q = (x2 , y2 ) y R = (x3 , y3 ) son tres puntos del plano,
entonces el área (P QR) del triángulo P QR viene dada por

1 1 1
1
(P QR) = x1 x2 x3 .
2
y1 y2 y3

A partir de aquı́, puede verse que si las coordenadas baricéntricas ho-


mogéneas P , Q y R respecto del triángulo ABC son P = (u1 : v1 : w1 ),
Q = (u2 : v2 : w2 ) y R = (u3 : v3 : w3 ), entonces,

u1 v1 w1

(u1 + v1 + w1 )(u2 + v2 + w2 )(u3 + v3 + w3 )(P QR) = u2 v2 w2 (ABC).
u3 v3 w3

o bien

u1 v1 w1

u2 v2 w2

(P QR) u3 v3 w3
= .
(ABC) (u1 + v1 + w1 )(u2 + v2 + w2 )(u3 + v3 + w3 )
56 TEMA 5. COORDENADAS BARICÉNTRICAS

5.4. Rectas
5.4.1. Ecuación de la recta que pasa por dos puntos
Teniendo en cuenta la fórmula del área del triángulo vista en la sección
anterior, la ecuación de la recta que une dos puntos con coordenadas ba-
ricéntricas homogéneas (u1 : v1 : w1 ) y (u2 : v2 : w2 ) vendrá dada por

u1 v1 w1

u2 v2 w2 = 0.

x y z

Problema 57. Hallar las ecuaciones de los lados del triángulo de referencia.
Solución. Las ecuaciones de los lados BC, CA y AB son, respectivamente,
x = 0, y = 0 y z = 0. Por ejemplo, como B = (0 : 1 : 0) y C = (0 : 0 : 1), la
recta BC tendrá por ecuación

0 1 0

0 0 1 = 0 ⇔ x = 0.

x y z

Problema 58. Hallar las ecuaciones de las mediatrices del triángulo de re-
ferencia.
Solución. La ecuación de la mediatriz de BC es (b2 − c2 )x + a2 (y − z) = 0.
En efecto, esta recta pasa por el punto medio de BC, con coordenadas (0 :
1 : 1) y por el circuncentro O de ABC, con coordenadas

a2 (b2 + c2 − a2 ) : b2 (c2 + a2 − b2 ) : c2 (a2 + b2 − c2 ).

Entonces, la ecuación de la mediatriz de BC es




2 2 02 1 1
a (b + c − a2 ) b2 (a2 + c2 − b2 ) c2 (a2 + b2 − c2 ) =0⇔

x y z
⇔ (a2 c2 − c4 − a2 b2 + b4 )x − a2 (b2 + c2 − a2 )(y − z) = 0 ⇔
⇔ (a2 c2 − c4 − a2 b2 + b4 )x + a2 (b2 + c2 − a2 )(y − z) = 0 ⇔
⇔ (b2 + c2 − a2 )((b2 − c2 )x + a2 (y − z)) = 0 ⇔
⇔ (b2 − c2 )x + a2 (y − z) = 0.

Problema 59. Hallar las ecuaciones de las bisectrices interiores del triángu-
lo de referencia.
5.4. RECTAS 57

Solución. La bisectriz interior del ángulo A es la recta que une el vértice


A = (1 : 0 : 0) y el incentro (a : b : c). Su ecuación será:

1 0 0

a b c = 0 ⇒ cy − bz = 0.

x y z

5.4.2. Puntos del infinito


Dado un triángulo ABC, tratemos de determinar el punto P cuyas coor-
denadas baricéntricas son, por ejemplo, (2 : 1 : −3). Lo intentamos hallando
el punto X sobre BC tal que BX : XC = −3 : 1 = 3 : −1, hallando el punto
Z sobre AB tal que AZ : ZB = 1 : 2 y haciendo la intersección de las rectas
AX y CZ:
A

B C X
Figura 26
Como resulta que AX y CZ son rectas paralelas, el punto P es infinito, es
el punto del infinito de estas dos rectas paralelas. En general, si u + v + w = 0
el punto con coordenadas (u : v : w) será un punto infinito.
El punto del infinito de la recta px + qy + rz = 0 es (q − r : r − p : p − q).
En efecto, este punto está en dicha recta, ya que cumple su ecuación, y es un
punto infinito, pues sus coordenadas suman cero.
Es evidente que x + y + z = 0 es la ecuación de la recta del infinito (del
plano del triángulo ABC).
Problema 60. Si P = (u1 : v1 : w1 ) y Q = (u2 : v2 : w2 ), siendo u1 +
v1 + w1 = u2 + v2 + w2 , resulta que el punto del infinito de la recta P Q es
∞P Q = (u1 − u2 , v1 − v2 , w1 − w2 ).
Solución. Por ser u1 +v1 +w1 = u2 +v2 +w2 , las coordenadas ∞P Q suman
cero, por lo que es un punto infinito. Además, tenemos que

u v w
1 1 1
u2 v2 w2 = 0,

u1 − u2 v1 − v2 w1 − w2
por lo que ∞P Q está en la recta P Q.
58 TEMA 5. COORDENADAS BARICÉNTRICAS

5.4.3. Rectas paralelas


La recta que pasa por P = (u : v : w) paralela a px + qy + rz = 0 tiene
por ecuación
q−r r−p p−q

u v w = 0.

x y z
Problema 61. Hallar las ecuaciones de las rectas paralelas por el punto
P = (u : v : w) a los lados del triángulo de referencia.
Solución. La recta BC tiene por ecuación x = 0, y su punto del infinito
es (0, 1, −1) (basta restar las coordenadas de B y C). La paralela a BC que
pasa por P = (u : v : w) es

0 1 −1

u v w = 0 ⇔ (v + w)x − u(y + z) = 0.

x y z

Las paralelas a CA y AB serán, respectivamente, (w + u)y − v(x + z) = 0 y


(u + v)z − w(x + y) = 0.

5.4.4. Intersección de dos rectas


La intersección de dos rectas
{
p1 x + q1 y + r1 z = 0,
p2 x + q2 y + r2 z = 0

es el punto ( )
q1 r1
: − p1 r1 : p1 q1 .
q2 r2 p2 r2 p2 q2
El punto del infinito de una recta l puede considerarse la intersección de l
con la recta del infinito x + y + z = 0.
Tres rectas pi x + qi y + ri z = 0, i = 1, 2, 3 son concurrentes si solo si

p1 q1 r1

p2 q2 r2 = 0.

p3 q3 r3

Problema 62. Sea DEF el triángulo medial de ABC, es decir, D, E, F son


los puntos medios de BC, CA, AB, respectivamente. Hallar la ecuación de
la recta que une el punto D con el excentro Ia = (−a : b : c). Análogamente,
5.5. PROBLEMAS DE LA EDICIÓN ANTERIOR 59

hallar las ecuaciones de las rectas que unen E con Ib y F con Ic . Demostrar
que estas tres rectas son concurrentes, hallando las coordenadas del punto
común.
Solución. La ecuación de la recta DIa es

x y z

0 1 1 = 0 ⇒ (b − c)x + ay − az = 0.

−a b c
Análogamente tendremos
EIb : −bx + (c − a)y + bz = 0.
F Ic : cx − cy + (a − b)z = 0.
Para demostrar que las tres rectas son concurrentes comprobamos que el
determinante formado por sus coeficientes se anula:

b−c a −a −c c b − a

−b c − a
b = −b c − a b = 0.

c −c a − b c −c a − b

(El segundo determinante es nulo, por tener dos filas proporcionales, y se


obtiene del primero sustituyendo la primera fila por la suma de las dos pri-
meras).
Para hallar el punto común a las tres rectas, resolvemos el sistema formado
por las dos primeras:
( )
a −a b − c −a b − c a
(x : y : z) = : − : =
c−a b −b b −b c − a
= (a(b + c − a) : b(a + c − b) : c(a + b − c)) =
= (a(s − a) : b(s − b) : c(s − c)) ,
que es el llamado Mittenpunkt.

5.5. Problemas de la edición anterior


Problema 63. Hallar las coordenadas baricéntricas de los puntos X, Y , Z
sobre los lados BC, CA, AB del triángulo ABC tales que

BX : XC = CY : Y A = AZ : ZB = 1 : 2.

Demostrar que el área triángulo formado por las rectas AX, BY y CZ es


1/7 del área del triángulo ABC.
60 TEMA 5. COORDENADAS BARICÉNTRICAS

Solución. El punto X sobre BC tal que BX : XC = 1 : 2 cumple que


(XBC) : (XCA) : (XAB) = 0 : 2 : 1, por lo que tenemos X = (0 : 2 : 1).
De la misma forma es Y = (1 : 0 : 2) y Z = (2 : 1 : 0). La recta AX
tiene ecuación y − 2z = 0, y lo mismo las rectas BY y CZ tienen ecuaciones
z − 2x = 0 y x − 2y = 0, respectivamente. El punto P de intersección de las
rectas BY y CZ es P = (2 : 1 : 4). De la misma forma, si Q = CZ ∩ AX y
R = AX ∩ BY , tenemos Q = (4 : 2 : 1) y R = (1 : 4 : 2).

2 1 4
(P QR) 1
= 4 2 1
(ABC) (2 + 1 + 4)(4 + 2 + 1)(1 + 4 + 2)
1 4 2

0 −7 0
1 7·7 1
= 3 0 −14 −7 = 3 = .
7 7 7
1 4 2

Problema 64. Comprobar que la recta que une los puntos medios de AB y
AC es paralela a la recta BC, obteniendo el punto de intersección de ambas
y viendo que es un punto del infinito.
Solución. El punto medio de AB es M = (1 : 1 : 0), y el punto medio de
AC es N = (0 : 1 : 1). La recta M N es x − y − z = 0, que corta a x = 0 en
(0 : 1 : −1), un punto del infinito, ya que sus coordenadas suman cero.

Problema 65. Hallar una condición sobre los lados a, b, c de un triángulo


para que la recta GI que une el baricentro con el incentro sea paralela al lado
BC.
Solución. La recta GI tiene ecuación:

1 1 1

a b c = 0 ⇒ (c − b)x + (a − c)y + (b − a)z = 0.

x y z

La intersección con x = 0 es el punto P = (0, b − a, c − a). Para que este


punto sea infinito, sus coordenadas deben sumar cero, por lo que debe ser
2a = b + c.

Problema 66. Dado el punto X = (0 : 1 − t : t) sobre BC, hallar un punto


Y sobre CA tal que la recta XY divida al triángulo ABC en dos partes de
igual área.
Solución. Dado el punto X = (0 : 1 − t : t) sobre BC, hallar un punto
Y sobre CA tal que la recta XY divida al triángulo ABC en dos partes de
igual área.
5.5. PROBLEMAS DE LA EDICIÓN ANTERIOR 61

Sea Y = (s : 0 : 1 − s) un punto sobre AC. Para que el área del triángulo


Y XC sea la mitad que la del triángulo ABC debe ser

s 0 1−s

0 t 1 − t = 1 ⇒ st = 1 ⇒ s = 1
2 2 2t
0 0 1
( )
1 1
⇒Y = :0:1− = (1 : 0 : 2t − 1).
2t 2t
Problema 67. Sean ABC un triángulo y P un punto de su plano. Por
P trazamos paralelas a AB, BC, CA, que cortan a BC, CA, AB en X,
Y , Z, respectivamente. Hallar las coordenadas baricéntricas de X, Y , Z en
términos de las de P y usarlas para razonar que
BX CY AZ
+ + = 1.
BC CA ZB

C'
O B'

B A' C
Figura 27

Solución. Si las coordenadas baricéntricas de O son u : v : w, lo que


representamos O = (u : v : w), entonces la paralela a AB por O tiene
ecuación
x y z

u v w = 0 ⇒ wx + wy − (u + v)z = 0.

1 −1 0
La intersección de esta recta con BC, de ecuación x = 0, es el punto A′ =
(0 : u + v : w). Entonces
BA′
BA′ BA′
w
A′ C u+v w
= = BA′
= = ,
BC BA′ + A′ C A′ C
+ 1 w
u+v
+ 1 u+v+w
62 TEMA 5. COORDENADAS BARICÉNTRICAS

y como, de igual forma, tenemos

CB ′ u AC ′ v
= , = ,
CA u+v+w AB u+v+w
al sumar obtendremos la igualdad propuesta, que es válida para cualquier
punto O, no necesariamente interior al triángulo, siempre que las distancias
se consideren con su correspondiente signo.

5.6. Problemas propuestos

5.1. Demostrar que son concurrentes las rectas que unen los vértices de un
triángulo con los puntos de contacto de la circunferencia inscrita y los lados
opuestos.
5.2. Sea D el punto medio del lado BC de un triángulo y M el punto
medio de la de la mediana AD. Supongamos que la recta BM corta a CA
en J. Establecer que 2A + B + C = 4M . Usarlo para hallar las proporciones
CJ : JA y BM : M J.
5.3. Sea ABC un triángulo y sean L, M, N puntos sobre BC, CA, AB,
respectivamente. Sean L′ , M ′ , N ′ los puntos simétricos de L, M, N respecto
de los puntos medios de los lados en los que se encuentran. Demostrar que
los triángulos LM N y L′ M ′ N ′ tienen la misma área.
5.4. Sea ABC un triángulo. Sean G el baricentro e Ia el excentro correspon-
diente al vértice A. Demostrar que las coordenadas baricéntricas del punto
simétrico A′ de A respecto de G son (−1 : 2 : 2). Usar esto para demostrar
que la recta Ia A′ será paralela al lado BC si y solo si b + c = 4a.
5.5. Demostrar que si P = (u : v : w) y Q = (u′ : v ′ : w′ ) son dos puntos
tales que u + v + w = u′ + v ′ + w′ entonces el punto medio de P y Q tiene
coordenadas (u + u′ : v + v ′ : w + w′ ).
Tema 6
Ecuaciones funcionales

6.1. Ecuaciones funcionales


Una ecuación funcional es una igualdad en la que la incógnita es una
función. Ejemplos de ecuaciones funcionales son:
f (f (x)) = f (x).

f (x + y) = f (x) + f (y).

f (x2 + f (y)) = y + f (x)2 .


En las ecuaciones más usuales se imponen condiciones a la función incóg-
nita para que la solución sea única, aunque no es obligatorio. Por ejemplo
se puede dar algún valor concreto, como f (0) = 0, o una propiedad de la
función, como que ésta es continua, derivable, monótona, etc.
Sabemos que una función no es sólo una fórmula. Una función también
lleva asociado un dominio, o conjunto de valores para los que está definida.

6.2. La ecuación f (x + y) = f (x) + f (y)


Comencemos por intentar hallar las funciones continuas f : R → R que
cumplen la llamada ecuación de Cauchy: f (x + y) = f (x) + f (y). Es un
ejemplo importante y la técnica usada en su resolución nos servirá para otros
casos.

Haciendo f (x) = ax + b, debe ser a(x + y) + b = (ax + b) + (ay + b),


es decir b = 0 y f es de la forma f (x) = ax. Veamos que estas son las
únicas soluciones.

Haciendo x = y = 0, la ecuación se reduce a f (0) = f (0) + f (0), por lo


que f (0) = 0.

63
64 TEMA 6. ECUACIONES FUNCIONALES

De f (2) = f (1) + f (1), deducimos que f (2) = 2 · f (1). Análogamente,


f (3) = f (2 + 1) = f (2) + f (1) = 3 · f (1) y, en general, podemos
demostrar que f (n) = nf (1). para cualquier natural n.

De 0 = f (0) = f (−n + n) = f (−n) + f (n), resulta que para cualquier


natural n es f (−n) = −f (n) = (−n) · f (1).

De n1 +· · ·+ n1 = 1 (teniendo la suma n sumandos), resulta que n·f ( n1 ) =


f (1), es decir f ( n1 ) = n1 · f (1).

De n1 + · · · + n1 = m n
(teniendo la suma m sumandos), resulta que
m
n
= m · f ( 1
n
) = m
n
· f (1).

Podemos concluir entonces, que f (r) = r · f (1) para cualquier número


racional r.

Si x es un número irracional, elegimos una sucesión {rn } de números


racionales cuyo lı́mite sea x. Entonces tendremos que

f (x) = lı́m f (rn ) = lı́m rn · f (1) = x · f (1).


n→∞ n→∞

En consecuencia, f (x) = ax para cualquier x, siendo a = f (1).


Conocida la solución de la ecuación de Cauchy podemos resolver otras
ecuaciones. Por ejemplo, hallemos las funciones continuas que cumplen la
ecuación f (x + y) = f (x)f (y).
Observemos primero que si una tal f se anula en un punto x0 , entonces,
para cualquier otro x es f (x) = f (x − x0 + x0 ) = f (x − x0 )f (x0 ) = 0.
Si f no se anula en ningún punto, podemos considerar la función g(x) =
ln |f (x)|, que por ser composición de funciones continuas, es una función
continua. Entonces, como se cumple

g(x + y) = ln |f (x + y)| = ln |f (x)f (y)|


= ln |f (x)| + ln |f (y)| = g(x) + g(y)

es g(x) = ax para un cierto a, y por tanto, f (x) = eax para un cierto a.

6.3. Técnicas de resolución


No hay un procedimiento general que sirva para resolver cualquier ecua-
ción funcional. Sin embargo, hay algunas técnicas que nos pueden ser útiles
para encontrar la solución o soluciones de una ecuación de este tipo.
6.4. PROBLEMAS RESUELTOS 65

¿Cuál es el dominio y recorrido de la función? Por ejemplo, si tenemos


una función f : N → N y f (mn) = f (m)f (n), haciendo m = n = 1
resultará que f (1) = 1. Si es f : Z → Z, también podrá ser f (1) = 0.

Podemos usar la ecuación funcional para obtener el valor de la función


en algún punto concreto. Por ejemplo, en la sección anterior hacı́amos
x = y = 0 en f (x + y) = f (x) + f (y) para obtener que f (0) = 0.

También puede ser de ayuda encontrar alguna función conocida que


cumpla la ecuación. Ası́, como hemos visto en el ejemplo anterior, si la
ecuación es f (x + y) = f (x)f (y), como sabemos que f (x) = ex cumple
dicha ecuación, es natural introducir la función g(x) = ln |f (x)|.

Exploraremos las simetrı́as en x e y, puede ayudar intercambiar los


papeles de x e y.

Deberemos usar adecuadamente las condiciones adicionales sobre la


función incógnita: continuidad, derivabilidad, monotonı́a, ...

6.4. Problemas resueltos


Problema 68. Sabiendo que f (x + y) = x + f (y) siendo x e y números
reales cualesquiera y que f (0) = 2, hallar f (2004).
Solución. Para cualquier x, f (x) = f (x + 0) = x + f (0) = x + 2. Por
tanto, f (2004) = 2006.

Problema 69. Hallar todas las funciones f : Z → Z, tales que para cuales-
quiera x, y enteros se verifica: f (x + f (y)) = f (x) − y.
Solución. Supongamos que existe una tal función f .
La función f es inyectiva ya que si x, y ∈ Z y f (x) = f (y) entonces,

0 + f (x) = 0 + f (y) ⇒ f (0 + f (x)) = f (0 + f (y)) ⇒


⇒ f (0) − x = f (0) − y ⇒ x = y.

Debe ser f (0) = 0, ya que

f (f (0)) = f (0 + f (0)) = f (0) − 0 = f (0) ⇒ f (0) = 0.

Ahora, f (f (x)) = −x para todo x ∈ Z, ya que

f (f (x)) = f (0 + f (x)) = f (0) − x = −x.


66 TEMA 6. ECUACIONES FUNCIONALES

También f (−x) = −f (x) para todo x ∈ Z, ya que

−x − f (x) =f (f (x + f (x)) = f (f (x) − x) =


f (−x + f (x)) = f (−x) − x ⇒ f (−x) = −f (x).

Entonces, si f (1) = w, entonces f (−1) = −w y f (−w) = 1 por .


Para cualquier n ∈ Z, f (n) = wn. Por ser f (−n) = −f (n), basta demos-
trarlo cuando n es positivo. Lo hacemos por inducción. Para n = 1 es cierto
y si f (n) = wn, entonces f (n + 1) = f (n + f (−w)) = f (n) + w = w(n + 1).
Por tanto, la fórmula es cierta para todo n.
Entonces, por un lado, f (w) = w2 , y por otro, f (w) = −1, y como
w2 = −1 es imposible si w ∈ Z, no existe ninguna función que cumpla la
condición del enunciado.
Problema 70. Hallar todas las funciones f : Q+ → Q+ tales que f (x + 1) =
f (x) + 1 y f (x3 ) = f (x)3 para cualquier número racional positivo x.
Solución. En primer lugar observamos que, por inducción, f (x + n) =
f (x)+n para cualquier número racional positivo x y cualquier número natural
n. ( )
Si q ∈ Q , llamamos t = f pq , y tenemos
p +

(( )3 ) ( )
p p3 2
f q
+q 2
=f q3
+ 3 pq2 q 2 +
+q =3 pq q 4 6

( )
p3
=f q3
+ 3p2 + 3pq 3 + q 6 = t3 + 3p2 + 3pq 3 + q 6 ,

y por otro lado,


(( )3 ) ( ( ))3 (
p
)3
f q
+q 2
= f pq + q 2 = t + q2 =

=t3 + 3q 2 t2 + 3q 4 t + q 6 .

Igualando, obtenemos una ecuación de segundo grado en t:


( 2 )
t2 + q 2 t − pq2 + pq = 0.

Como t = pq es la única solución positiva de esta ecuación, resulta que f (x) =


x para cualquier x ∈ Q+ .
Problema 71. Hallar todas las funciones f : R − {0, 1} → R tales que
( )
1
f (x) + f = x.
1−x
6.4. PROBLEMAS RESUELTOS 67

Solución. Para resolver este problema hacemos repetidamente la sustitu-


ción x → 1−x
1
, obteniendo
 ( )

 1

 f (x) + f = x,

 1−x

 ( 1 ) ( )
x−1 1
f +f = ,

 1−x x 1−x

 ( )

 x−1 x−1

f + f (x) = ,
x x

y restando la segunda desigualdad a la tercera resulta


 ( )
 1

 f (x) + f 1 − x = x,
( )

 1 x−1 1 −x2 + x − 1
 f (x) − f = − = ,
1−x x 1−x x − x2

de donde ( )
1 x2 − x + 1 x3 − x + 1
f (x) = x+ = .
2 x2 − x 2x2 − 2x

Problema 72. Demostrar que sólo existe una función estrictamente cre-
ciente f : N → N tal que f (2) = 2 y f (mn) = f (m)f (n) para cualesquiera
m, n ∈ N.
Solución. En primer lugar, debe ser f (1) = f (1)f (1), por lo que f (1) = 1.
Además, de f(2)=2 se deduce fácilmente, por inducción que f (2k ) = 2k para
cualquier k > 1. Esto indica que quizá sea f (n) = n para cualquier n.
Demostrémoslo por inducción. Supongamos que n > 1 y que que f (j) = j
para todos los j menores que n. Entonces, si n es par, podemos expresar
n = 2j para algún j, tendremos f (n) = f (2j) = f (2)f (j) = 2j = n. Y si n
es impar, podemos expresar n = 2j + 1, y tendremos

2j = f (2j) < f (2j + 1) < f (2j + 2) = f (2(j + 1)) = 2(j + 1) = 2j + 2.

Entonces 2j < f (2j + 1) < 2j + 2 ⇒ f (2j + 1) = 2j + 1 ⇒ f (n) = n.

Problema 73. Hallar todas las f : R → R tales que

f (x2 − y 2 ) = (x − y) (f (x) + f (y)) .


68 TEMA 6. ECUACIONES FUNCIONALES

Solución. Intercambiando los papeles de x e y, f (y 2 −x2 ) = (y −x)(f (x)+


f (y)) = −f (x2 − y 2 ), y como x, y varı́an en R, podemos deducir que f (−x) =
−f (x) para cualquier x. Ahora, cambiando y por −y,
(x − y)(f (x) + f (y)) =f (x2 − y 2 ) = f (x2 − (−y)2 )
=(x + y)(f (x) + f (−y)) = (x + y)(f (x) − f (y)),
de donde, simplificando, xf (y) = yf (x), y haciendo y = 1, resulta que f (x) =
f (1)x para cualquier x ∈ R.

6.5. Problemas de la edición anterior


Problema 74. Asignamos un entero no negativo f (n) a cada entero positivo
n, cumpliéndose las siguientes condiciones:
1. f (mn) = f (m) + f (n) para cualesquiera enteros positivos m y n.
2. f (n) = 0 si el desarrollo decimal de n acaba en 3.
3. f (10) = 0.
Demostrar que f (n) = 0 para cualquier n.
Solución. Teniendo en cuenta la primera condición, es suficiente resolver
el problema en el caso de que el número n sea primo. Consideramos los
diferentes casos según la cifra en que acabe el número primo n. Si n acaba
en 3, la segunda condición garantiza que f (n) = 0. De la segunda condición
obtenemos que f (2)+f (5) = f (2·5) = f (10) = 0, por lo que f (2) = f (5) = 0,
y queda probar que f (n) = 0 si n es un número primo que acaba en 1, 7 o 9.
Si n acaba en 1, 3n acaba en 3 y f (n) = f (n) + f (3) = f (3n) = 0.
Si n acaba en 7, 9n acaba en 3 y evidentemente es f (9) = f (3)+f (3) =
0, por lo que también tenemos f (n) = f (n) + f (9) = f (9n) = 0.
Si n acaba en 9, 7n acaba en 3 y f (n) = f (n) + f (7) = f (7n) = 0.
Problema 75. Hallar todas las funciones f : R → R tales que f (a + x) −
f (a − x) = 4ax para cualesquiera a, x ∈ R.
Solución. Teniendo en cuenta que
(x x) (x x) x x
f (x) − f (0) = f + −f − = 4 · · = x2 ,
2 2 2 2 2 2
vemos que f (x) = a + x2 con a = f (0), y recı́procamente, todas todas las
funciones de esta forma son evidentemente soluciones.
6.5. PROBLEMAS DE LA EDICIÓN ANTERIOR 69

Problema 76. Demostrar que no existe ninguna función f : N → N estric-


tamente creciente tal que f (2) = 3 y f (mn) = f (m)f (n) para cualesquiera
m, n ∈ N.
Solución. Razonamos por reducción al absurdo. Si una tal función f
existiera, serı́a f (an ) = f (a)n para cualesquiera a, n ∈ N. En particular,
f (3)3 = f (27) < f (32) = f (2)5 = 35 = 243 ⇒ f (3) 6 6. Por otro lado,
f (3)2 = f (9) > f (8) = f (2)3 = 33 = 27, por lo que f (3) > 6. En consecuen-
cia debe ser f (3) = 6.
De la misma forma, vamos a determinar el valor f (5). Por un lado es
f (5)3 = f (125) < f (127) = f (27 ) = 37 = 2187 < 2197 = 133 , por lo que
f (5) 6 12. Por otro lado tenemos f (5)4 = f (625) > f (512) = f (29 ) = 39 >
114 , por lo que f (5) > 12, ası́ que tenemos f (5) = 12.
Pero los valores obtenidos son incompatibles, ya que f (10) = f (2)f (5) =
3 · 12 = 36, y también f (9) = f (3)2 = 62 = 36, por lo que f no serı́a
estrictamente creciente.

Problema 77. Determinar las funciones reales f definidas y derivables en


toda la recta real, que satisfacen la ecuación funcional:
( )
x+y
f (f (x) + f (y)) = 2f (x)f (y)
2

para todo x, y ∈ R y siendo f (x) ̸= 0 para todo x ∈ R.


Solución. Por ser f (x) ̸= 0 para todo x ∈ R, podemos considerar la
función derivable g : R → R, g(x) = 1/f (x). Para todo x, y tenemos que
( ) ( )
x+y 1 f (x) + f (y) 1 1 1 g(x) + g(y)
g = ( x+y ) = = + = .
2 f 2 2f (x)f (y) 2 f (x) f (y) 2

Considerando y constante y derivando respecto de x,


( )
′ x+y g ′ (x)
g = ,
2 2

y considerando x = −y obtenemos g ′ (x) = 0, por lo que la función g es cons-


tante y f también lo es. Por tanto, las funciones que cumplen el enunciado
son las funciones constantes no nulas.

Problema 78. Hallar todas las funciones f : R − {−1} → R tales que


( )
1−x
2f (x) − f = x − 1.
1+x
70 TEMA 6. ECUACIONES FUNCIONALES

Solución. Haciendo la sustitución x → 1−x


1+x
,
 ( )
 1−x

 2f (x) − f 1 + x = x − 1,
( )

 1−x 1−x −2x
 2f − f (x) = −1=
1+x 1+x 1+x

y entonces, sumando a la segunda igualdad el doble de la primera, tenemos

2x 2(x2 − x − 1) 2(x2 − x − 1)
3f (x) = 2(x − 1) − = ⇒ f (x) = .
1+x 1+x 3x + 3

6.6. Problemas propuestos

6.1. Hallar todas las f : R → R tales que f (x + f (y)) = x + f (f (y)) y


f (2008) = 2009.
6.2. Hallar todas las funciones f : R → R tales que
2f (x) + f (1 − x) = x2 ∀x ∈ R

6.3. Sea f : N → N una función estrictamente creciente que cumple


f (f (n)) = 3n para cualquier número natural n. Hallar f (2009).
6.4. Hallar todas las funciones f : N → N tales que f (f (n)) + f (n) = 2n
para todo n.
6.5. Hallar todos los polinomios P (x) con coeficientes enteros tales que
P (P ′ (x)) = P ′ (P (x)).
Apéndice A
Soluciones

Introducción (página 17)


1.1. Está claro que P (1) es cierta. Lo que debe fallar es la implicación

P (n) es cierta ⇒ P (n + 1) es cierta.

Observemos que cuando n = 1 los grupos formados por las personas p1 , . . . , pn


y p2 , . . . , pn+1 son exactamente {p1 } y {p2 }, que no tienen ninguna persona
en común. Por tanto, la implicación falla cuando n = 1.
1.2. Si 7 divide a a+b y a a2 +b2 , también dividirá a 2ab = (a+b)2 −(a2 +b2 ).
Entonces 7 divide a alguno de los números a y b. Si, por ejemplo, 7|a, entonces
7 también divide a b = (a + b) − a.
1.3. Hemos de demostrar que un número es divisible por 11 si y solo si la
diferencia de las sumas alternadas de sus cifras es divisible por 11. Dado el
número N = an an−1 . . . a1 a0 , tenemos
N = a0 + 10a1 + 100a2 + · · · + 10n an ,
S = a0 − a1 + a2 + · · · + (−1)n an ,
y, restando, N − S = 11a1 + 99a2 + · · · + (10n − (−1)n ) an .
Serı́a suficiente probar que 10k − (−1)k es siempre divisible por 11. Lo
probamos por inducción. Para k = 1 tenemos 10 − (−1) = 11, y suponiendo
que 10k − (−1)k es divisible por 11, tenemos que
( )
10k+1 − (−1)k+1 = 10 · 10k + (−1)k =10 · 10k − (−1)k + 10 · (−1)k + (−1)k
( )
=10 · 10k − (−1)k + 11 · (−1)k
también es divisible por 11.
1.4 Usaremos el principio del palomar. Consideremos los nichos formados
por los dı́as consecutivos 1◦ − 2◦ , 2◦ − 3◦ , . . . , 11◦ − 12◦ , en total 11 nichos.

71
72 APÉNDICE A. SOLUCIONES

Llamemos x1 , . . . , x12 a las horas en que el ordenador ha sido usado los dias
1, . . . , 12. Tenemos x1 = 7, x12 = 6 y x2 + · · · x11 = 99 − 13 = 86. El tiempo
consumido por los 11 nichos es T = x1 + 2(x2 + · · · + x11 ) + x12 = 2 · 86 + 13 =
185 = 11 · 17 horas, por lo que a algún nicho corresponden 17 horas o más.
1.5. Razonamos por inducción sobre n. Llamando an = xn + x1n , a1 es entero
por hipótesis, y si a1 , a2 , . . . , an son enteros, teniendo en cuenta que
( )( )
n 1 1 1 1
x + n x+ =xn+1 + xn−1 + n−1 + n+1 =
x x x x
( ) ( )
n+1 1 n−1 1
= x + n+1 + x + n−1 ,
x x

resulta que an+1 = an a1 − an−1 también es entero.

Simetrı́a (página 26)

2.1. Usando la relación cos(180◦ − α) = − cos α y teniendo en cuenta que


cos 90◦ = 0, tenemos

cos 1◦ + cos 2◦ + · · · + cos 178◦ + cos 179◦




90
= (cos k + cos(180◦ − k))
k=1


90
= 0 = 0.
k=1

2.2. Desarrollando y agrupando tenemos que

(x + y + z)3 =x3 + y 3 + z 3 + 3(x2 y + x2 z + y 2 x + y 2 z + z 2 x + z 2 y) + 6xyz


=x3 + y 3 + z 3 + 3xy(x + y) + 3xz(x + z) + 3yz(y + z) + 6xyz
=x3 + y 3 + z 3 + 3xy(x + y + z) + 3xz(x + z + y) + 3yz(y + z + x)
− 9xyz + 6xyz
=x3 + y 3 + z 3 + 3(x + y + z)(xy + yz + zx) − 3xyz,

de donde resulta que x3 + y 3 + z 3 = σ13 − 3σ1 σ2 + 3σ3 .


2.3. El sistema es simétrico respecto de las tres variables x, y, z, por lo que
podemos esperar que la solución sea x = y = z. Como x = 3 es una solución
de la ecuación x3 − 9x2 + 27x − 27 = 0, obtenemos la solución x = y = z = 3.
73

Para demostrar que no hay más soluciones, observamos por ejemplo que
x3 − 9x2 + 27x + 27 = (x − 3)3 , por lo que el sistema se puede expresar
 3
 y + (x − 3) − x = 0
 3 3

z 3 + (y − 3)3 − y 3 = 0 ,

 3
x + (z − 3)3 − z 3 = 0

y sumando las tres ecuaciones obtenemos (x − 3)2 + (y − 3)2 + (z − 3)2 = 0,


que sólo puede ser si x = y = z = 3.
2.4. La figura siguiente muestra un triángulo ABC, cuyas tres bisectrices se
cortan en el incentro I.

Z' Y'

Y
Z I

B X C
Figura 28

Dado un punto X, su simétrico Z respecto BI estará sobre la recta BA


y el simétrico Y de Z respecto de la bisectric AI estará sobre la recta AC.
De la misma forma, dada el punto X, el simétrico Y ′ respecto de la recta
CI está sobre CA y el simétrico Z ′ de Y ′ respecto de AI está sobre AB. En
resumen, si son dados el punto X y las rectas IA, IB e IC, podemos hallar
las rectas Y Y ′ y ZZ ′ , que serán los lados AC y AB del triángulo buscado.
La recta Y Y ′ encontrará a la recta IC en C la recta ZZ ′ cortará a la recta
IB en B.
2.5. Llamemos P ′ al punto simétrico de P respecto de la recta dada r. Sea X
un punto variable sobre la recta r. Los triángulos equiláteros P XY y P XZ
tienen por vértices Y , Z a los puntos de intersección de las circunferencias
con centros P y X y radio P X.
74 APÉNDICE A. SOLUCIONES

r
P'

Figura 29

Entonces, el ángulo inscrito Y P ′ P es la mitad del ángulo central Y XP ,


que mide 60◦ , por tanto ∠Y P ′ P = 30◦ , y el punto Y está sobre una recta
que forma un ángulo de 30◦ con la recta P P ′ . De la misma dorma, el punto
Z está sobre otra recta que forma con P P ′ un ángulo de 30◦ , simétrica de la
anterior, respecto del eje P P ′ .

Desigualdades (página 36)


3.1. Usaremos que (x − a)(x − b) 6 0 si y solo si a 6 x 6 b. En nuestro caso
tenemos
(m ) ( 10n + m )
m − 2n 10n + m − 2m − 8n
−2 −2 = ·
n m + 4n n m + 4n
m − 2n −m + 2n (m − 2n)2
= · =− 6 0.
n m + 4n n(m + 4n)

3.2. Usando los triángulos semejantes AXY ∼ Y XB tenemos que


AX YX
= ⇒ AX · XB = XY 2 ,
XY XB
por lo que XY es la media aritmética de AX y XB.
75

C
Y

A O X B

Figura 30

En la figura, hemos trazado el radio OC = r. Tenemos que


√ AB AX + XB
AX · XB = XY 6 OC = r = = ,
2 2
es decir, la media geométrica es siempre menor o igual que la media aritmé-
tica.
3.3. La desigualdad de Cauchy-Schwarz nos dice que
√ √ √
x+y y+z z+x
+ +
x+y+z x+y+z x+y+z
√ √ √
x+y y+z z+x
=1 · +1· +1·
x+y+z x+y+z x+y+z

√ x+y y+z z+x
6 12 + 12 + 12 · + +
x+y+z x+y+z x+y+z

√ 2(x + y + z) √
6 3· = 6.
3(x + y + z)

3.4. Usamos primero la desigualdad de Chebyshev y luego la desigualdad de


las medias armónica y geométrica:
( ) ( )
1 b+c c+a a+b 1 1 1 1 1
+ + > · ((b + c) + (c + a) + (a + b)) · + +
2 a b c 2 3 a b c
(1 1 1)
+ + a+b+c
=(a + b + c) · a b c > √ 3
3 abc.
76 APÉNDICE A. SOLUCIONES

3.5. Usando la desigualad entre las medias armónica y aritmética, y la de-


sigualdad a2 + b2 + c2 > ab + bc + ca,

1 1 1 9
+ + >
1 + ab 1 + bc 1 + ca (1 + ab) + (1 + bc) + (1 + ca)
9 3
> 2 2 2
= .
3 + (a + b + c ) 2

Geometrı́a del triángulo (página 49)

4.1. Usando la fórmulas del ángulo mitad


√ √ √
b2 +c2 −a2
A 1 − cos A 1− 2bc a2 − (b − c)2
sen = = =
2 2 2 4bc
√ √
(a − b + c)(a + b − c) (s − b)(s − c)
= = .
4bc bc

De la misma forma obtenemos


√ √ √
1 + b +c2bc−a
2 2 2
A 1 + cos A (b + c)2 − a2
cos = = =
2 2 2 4bc
√ √
(a + b + c)(b + c − a) s(s − a)
= = .
4bc bc

La fórmula de la tangente se obtiene fácilmente dividiendo las dos anteriores.


4.2. BI y BIa son las bisectrices interior y exterior del ángulo B, por lo que
aplicando el teorema de la bisectriz al triángulo ABL,

C
Ia

L
I

A B
Figura 31
77

LI BL LIa
= =−
IA BA Ia A
AI IA LI
⇒ = =− .
AIa Ia A LIa
4.3. Usando el concepto de potencia de un punto respecto de una circunfe-
rencia, BD · BD′ = BF · BF ′ , CE · CE ′ = CD · CD′ y AF · AF ′ = AE · AE ′ ,
por lo que obtenemos
BD′ CE ′ AF ′ BD′ CE ′ AF ′
· · = · ·
D′ C E ′ A F ′ B F ′ B D′ C E ′ A
F B DC EA 1 1
= · · = = = 1.
BD CE AF DC
BD
· EA
CE
· FB
AF
1

4.4. Usando el teorema de la bisectriz sabemos que BL y LC son proporcio-


nales a AB y AC, respectivamente. Si escribmos
BL LC
k= = ,
AB AC
podemos obtener
a
a = BC = BL + LC = k · AB + k · AC = k · (b + c) ⇒ k = ,
b+c
y esto nos permite obtener
ac ab
BL = , LC = .
b+c b+c
Entonces
1 1 2
2
+ 2
=
BL LC AD2
2
(b + c) (b + c)2 2 2(b + c)2
⇒ 2 2 + = ( ) =
ca a 2 b2 bc 1 − (b+c)
a2 bc(b + c)2 − a2 bc
2

b 2 + c2 2
⇒ =
2
abc 2 2 bc(b + c)2 − a2 bc
⇒bc(b2 + c2 )(b + c)2 − a2 bc(b2 + c2 ) = 2a2 b2 c2
bc(b2 + c2 )(b + c)2 bc(b2 + c2 )(b + c)2
⇒a2 = 2 2 = = b 2 + c2 ,
2b c + bc(b2 + c2 ) bc(2bc + b2 + c2 )
y por el teorema de Pitágoras, el ángulo A debe ser recto.
78 APÉNDICE A. SOLUCIONES

4.5. Hay que demostrar que se cumple la igualdad


BD CE AF
· · = −1.
DC EA F B
Observemos que los triángulos CAD y ABD son semejantes. Por un lado,
el ángulo D es común. Por otro lado, el ángulo inscrito DCA abarca el mismo
arco AB que interceptan los lados del ángulo semiinscrito DAB. Entonces,
los ángulos DCA y DAB también son iguales. Entonces podmeos calcular:

BD BD BD2 BD2 AB 2 c2
=− =− =− = − = − .
DC CD BD · CD AD2 CA2 b2
De igual forma,
CE a2 AF b2
= − 2, = − 2.
EA c FB a
Y multiplicando las tres igualdades,
( 2) ( 2) ( 2 )
BD CE AF c a b
· · = − 2 · − 2 · − 2 = −1.
DC EA F B b c a

Coordenadas baricéntricas (página 62)


5.1. Los puntos de contacto de la circunferencia inscrita con los lados BC,
CA, AB son X = (0 : s−c : s−b), Y = (s−b : 0 : s−a), Z = (s−b : s−c : 0).
La recta AX tiene ecuación (s − b)y − (s − c)z = 0, ya que A = (1 : 0 : 0)
y X = (0 : s − c : s − b) cumplen evidentemente esta ecuación. Entonces lo
que tenemos que comprobar es que se anula el determinante

0 s − b −(s − c)

∆ = −(s − a) 0 s − c .
s−a −(s − b) 0

Llamando λ = (s − a)(s − b)(s − c) tenemos



0 1 −1 0 1 −1 0 1 −1


∆ = λ −1 0
1 = λ −1 0 1 = λ −1 0 1 = 0.

1 −1 0 0 −1 1 0 0 0

5.2. Por ser D el punto medio del lado BC tenemos que 2D = B + C y


por ser M el punto medio de la mediana AD, es 2M = A + D. Entonces,
4M = 2A + 2D = 2A + B + C. Escribiendo esta igualdad en la forma
79

4M = B + (2A + C). Llamando J ′ al punto sobre CA tal que 3J ′ = 2A + C,


es decir tal que CJ ′ : J ′ A = 2 : 1, resulta que J ′ , B y M están alineados, por
lo que J ′ = J y ası́ tenemos CJ : JA = 2 : 1 y BM : M J = 3 : 1.
5.3. En general, si X = (0 : u : u′ ) es un punto sobre BC, su simétrico
respecto del punto medio es X ′ = (0 : u′ : u). Llamemos L = (0 : u : u′ ),
M = (v ′ : 0 : v), N = (w : w′ : 0), y ası́ tendremos L′ = (0 : u′ : u),
M ′ = (v : 0 : v ′ ), N ′ = (w′ : w : 0). Aplicando la fórmula del área del
triángulo,
0 u u′
(LM N ) ′
= v 0 v = u′ v ′ w′ + uvw,
(ABC) w w′ 0
una expresión invariante al hacer el cambio de variable

(u → u′ , v → v ′ , w → w′ ),

por lo que el mismo valor se obtendrá al calcular el área del triángulo


(L′ M ′ N ′ ).
5.4. Teniendo en cuenta que 3G = A + B + C y que 2G = A + A′ , podemos
escribir 2A + 2B + 2C = 6G = 3A + 3A′ ⇒ 3A′ = −A + 2B + 2C. Para hallar
la ecuación de la recta Ia A′ calculamos el determinante

x y z

−1 2 2 = (2c − 2b)x + (c − 2a)y − (b − 2a)z = 0.

−a b c

Esta recta corta a la recta BC(x = 0) en el punto (0 : b − 2a : c − 2a), que


será un punto del infinito si y solo si 0 + (b − 2a) + (c − 2a) = 0, es decir si
b + c = 4a.
5.5. Si P = (u : v : w) y Q = (u′ : v ′ : w′ ), se cumple (u + v + w)P =
uA + vB + wC y (u′ + v ′ + w′ )P = u′ A + v ′ B + w′ C, llamando al M al punto
medio de P y Q tendremos P + Q = 2M y

(u + u′ )A + (v + v ′ )B + (w + w′ )C = (uA + vB + wC) + (u′ A + v ′ B + w′ C)


=(u + v + w)P + (u′ + v ′ + w′ )Q = (u + v + w)(P + Q) = (u + v + w)(2M )
=(2u + 2v + 2w)(M ) =
= ((u + u′ ) + (v + v ′ ) + (w + w′ )) M.

De aquı́, M = (u + u′ : v + v ′ : w + w′ ).
Otra forma de verlo es tener en cuenta que si M es el punto medio de P Q,
será 2(M BC) = (P BC) + (QBC), y lo mismo para los otros lados. Entonces
80 APÉNDICE A. SOLUCIONES

podemos expresar
{
(P BC) : (P CA) : (P AB) = u : v : w
(QBC) : (QCA) : (QAB) = u′ : v ′ : w′
{
((P BC), (P CA), (P AB)) = (ku, kv, kw)

((QBC), (QCA), (QAB)) = (k ′ u′ , k ′ v ′ , k ′ w′ )
 ′ ′

 2(M BC) = (P BC) + (QBC) = ku + k u
⇒ 2(M CA = (P CA) + (QCA) = kv + k ′ v ′

 2(M AB) = (P AB) + (QAB) = kw + k ′ w′ .

Ahora, siendo u + v + w = u′ + v ′ + w′ el sistema en k, k ′ , k ′′


 ′ ′ ′′ ′

 ku + k u =k (u + u )
kv + k ′ v ′ =k ′′ (v + v ′ )

 kw + k ′ w′ =k ′′ (w + w′ )

admite la solución k ′′ = 21 (k + k ′ ), lo que nos permite escribir

M = (2(M BC) : 2(M CA) : 2(M AB))


= (ku + k ′ u′ : kv + k ′ v ′ : kw + k ′ w′ )
= (k ′′ (u + u′ ) : k ′′ (v + v ′ ) : k ′′ (w + w′ ))
= (u + u′ : v + v ′ : w + w′ ) .

Ecuaciones funcionales (página 70)


6.1. Haciendo y = 0 tenemos f (x + f (0)) = x + f (f (0)) y por tanto, para
cualquier x
f (x) = f ((x − f (0)) + f (0)) = x − f (0) + f (f (0)),
es decir, f (x) = x + a para cierta constante a. Como sabemos que f (2008) =
2009, tenemos que 2009 = 1 + a y a = 1, luego f (x) = x + 1 para todo x.
6.2. Haciendo el cambio de x por 1 − x resulta
{ {
2f (x) + f (1 − x) = x2 2f (1 − x) + 4f (x) = 2x2

2f (1 − x) + f (x) = (1 − x)2 2f (1 − x) + f (x) = (1 − x)2
⇒3f (x) = 2x2 − (x2 − 2x + 1) = x2 + 2x − 1
x2 + 2x − 1
⇒f (x) = .
3
81

6.3. No puede ser f (1) = 1 ya que entonces serı́a f (f (1)) = f (1) = 1 en


contra de la hipótesis que indica que f (f (1)) = 3 · 1 = 3. Además, si fuera
f (1) > 3, tendrı́amos

3 = f (f (1)) > f (3) > f (1) > 3,

que es imposible. Entonces debe ser f (1) = 2. A partir de aquı́ tenemos que
f (2) = f (f (1)) = 3, f (3) = f (f (2)) = 6, y ası́, también f (6) = f (f (3)) = 9.
En general, podemos ver fácilmente que f (3n ) = 2 · 3n y que f (2 · 3n ) = 3n+1 .
En efecto, ya hemos probado el caso n = 1 para ambas fórmulas, ya que
f (3) = 6 y f (6) = 9. Suponiendo que ambas fórmulas son ciertas para n,
tenemos,
f (3n+1 ) = f (f (2 · 3n )) = 3 · (2 · 3n ) = 2 · 3n+1 ,
f (2 · 3n+1 ) = f (f (3n+1 )) = 3 · 3n = 3n+2 ,
por lo que ambas fórmulas son ciertas para n + 1.
Ahora observemos que entre 3n y 2 · 3n hay 3n − 1 números, los mismos
que hay entre 2 · 3n y 3n+1 . Siendo la función f estrictamente creciente, debe
ser f (3n + k) = 2 · 3n + k para todos los k = 1, . . . , 3n − 1. Para acabar de
obtener todos los valores de la función f , observamos que f (2 · 3n + k) =
f (f (3n + k)) = 3 · (3n + k) = 3n+1 + 3k para todos los k = 1, · · · , 3n − 1.
En particular, como 2009 = 2·36 +551, será f (2009) = 37 +3·551 = 3840.
6.4. Es claro que f (n) = n es una solución. Veamos que es la única posible.
Si hacemos n = 1, tenemos f (1) + f (f (1)) = 2, de donde f (f (1)) = f (1) = 1.
Razonando por inducción, supongamos que para un cierto n > 1 es f (k) = k
para todo k 6 n. Entonces debe ser f (m) > n para todo m > n. Por tanto
será
f (n + 1) + f (f (n + 1)) > (n + 1) + (n + 1) = 2(n + 2).
Por hipótesis, tenemos exactamente que

f (n + 1) + f (f (n + 1)) = (n + 1) + (n + 1) = 2(n + 2).

Entonces resulta que f (f (n + 1)) = f (n + 1) = n + 1, y queda probado por


inducción que f (n) = n para todo n.
6.5. Si P (x) = ax + b, entonces P ′ (x) = a y tenemos

P (P ′ (x)) = a2 + b = a = P ′ (P (x)) ⇒ b = a − a2 ⇒ P (x) = ax + a − a2 .

Si P (x) es un polinomio de grado mayor que 2,



n ∑
n
P (x) = ak xk ⇒ P ′ (x) = kak xk−1 ,
k=0 k=1
82 APÉNDICE A. SOLUCIONES

ası́ que tenemos


( )k

n ∑
n
P (P ′ (x)) = ak kak xk−1 ,
k=0 k=1

cuyo término de mayor grado es xk(k−1) y su coeficiente es an · (nan )n =


nn an+1
n , mientras que
( )k−1

n ∑
n
P ′ (P (x)) = kak ak xk ,
k=1 k=0

que también es del mismo grado, pero ahora el coeficiente es nan (an )n−1 =
nann . Siendo an ̸= 0, obtenemos
1
nn an+1
n = nann ⇒ nn−1 an = 1 ⇒ an = ,
nn−1
lo cual es imposible en el caso de n > 2 y n entero.
Para saber más...

[1] Societat Catalana de Matemàtiques, Sessions de preparació per a


l’Olimpı́ada Matemàtica. Barcelona, 2002.

[2] H.S.M. Coxeter, S. L. Greitzer: Retorno a la Geometrı́a. (Traducción de


Geometry Revisited.) DLS-Euler Editores. Madrid.

[3] Howard Eves: A Survey of Geometry. Allyn and Bacon. Boston.

[4] Larson, Loren C., Problem-solving through problems. Springer, 1973.

[5] V. Lidski y otros: Problemas de Matemáticas Elementales. Editorial


MIR. Moscú.

[6] Marie, Frère Gabriel: Exercises de géométrie, comprenant l’esposé des


méthodes géométriques et 2000 questions résolues. A. Mame e hijos.
Tours, 1912.

[7] George Polya: How solve it. A New Aspect of Mathematical Method.
Segunda Edición. Penguin Books. Londres.

[8] Levi S. Shively: Introducción a la Geometrı́a Moderna. CECSA. Mexico.

[9] Paul Y. Yiu: An Introduction to the Geometry of the Triangle, Florida,


2001. Disponible en http : //www.math.f au.edu/yiu/geometry.html.

83

También podría gustarte